You are on page 1of 80

Criminal Law Review- Midterm Consolidated Cases II.

Criminal liability of each accused-appellant; As a


Principal; As an Accomplice

there is no doubt that Petrus is liable as principal of


PEOPLE v PETRUS YAU AND SUSANA YAU y the crime of kidnapping for ransom. Susana, on the other
SUMOGBA; G.R. No. 208170; August 20, 2014 hand, is liable only as an accomplice to the crime as correctly
found by the lower courts. It must be emphasized that there
was no evidence indubitably proving that Susana
participated in the decision to commit the criminal act.
I. The elements of Kidnapping For Ransom under
Article 267 of the RPC, as amended by R.A. No. 7659, Jurisprudence is instructive of the elements required, in
are as follows: accordance with Article 18 of the RPC, in order that a person
may be considered an accomplice, namely;
(a) intent on the part of the accused to deprive the
victim of his liberty; (1) that there be a community of design; that is,
knowing the criminal design of the principal
(b) actual deprivation of the victim of his liberty; and by direct participation, he concurs with the
latter in his purpose;
(c) motive of the accused, which is extorting ransom
for the release of the victim. (2) that he cooperates in the execution by
previous or simultaneous act, with the
All of the foregoing elements were duly established intention of supplying material or moral aid in
by the testimonial and documentary evidences for the the execution of the crime in an efficacious
prosecution in the case at bench. First, Petrus is a private way; and
individual. Second, Petrus kidnapped Alastair by using
sleeping substance which rendered the latter unconscious (3) that there be a relation between the acts done
while inside a taxicab driven by the said accused-appellant. by the principal and those attributed to the
Third, Petrus took and detained Alastair inside the house person charged as accomplice.
owned by him and Susana Yau in Bacoor, Cavite, where said
victim was handcuffed and chained, and hence, deprived of In the case at bench, Susana knew of the criminal design of
his liberty. Fourth, Alastair was taken against his will. And her husband, Petrus, but she kept quiet and never reported
fifth, Petrus made demands for the delivery of a ransom in the incident to the police authorities. Instead, she stayed
the amount of US$600,000.00 for the release of the victim. with Petrus inside the house and gave food to the victim or
accompanied her husband when he brought food to the FACTS:
victim. Susana not only countenanced Petrus illegal act, but
also supplied him with material and moral aid. It has been 1. Petrus and Susana were charged with the crime of
held that being present and giving moral support when a Kidnapping For Ransom of private complainant
crime is being committed make a person responsible as an Alastair Onglingswam, who was put into sleep while
accomplice in the crime committed. As keenly observed by riding a taxi (White Toyota taxi cab with plate
the RTC, the act of giving food by Susana to the victim was number PVD-115) bound to San Juan, Manila.
not essential and indispensable for the perpetration of the Subsequently, Chau (Alastair girlfriend) received an
crime of kidnapping for ransom but merely an expression of email from the purported kidnapper demanding
sympathy or feeling of support to her husband. Moreover, US$2,000.00;
this Court is guided by the ruling in People v. De Vera, where
it was stressed that in case of doubt, the participation of the 2. During twenty-two (22) days of captivity, he was
offender will be considered as that of an accomplice rather allowed to communicate with his family almost daily
than that of a principal. to prove that he was still alive and was served with
meals almost five times a day either by Petrus or the
III. Penalty other accused Susan Yau;

With respect to the penalty, the Court finds that the RTC was 3. The police were able to chance upon the said vehicle.
correct in imposing the penalty of reclusion perpetua they followed it, then flagged it down and
without eligibility of parole against Petrus as approached the driver. The driver was asked to scroll
principal in the charge of kidnapping for ransom in view of down his window and was told that the vehicle was
R.A. No. 9346, prohibiting the death penalty. Also, the Court being used to victimize foreign nationals. Appellant
finds that the penalty of eight (8) years and one (1) day did not offer to make any comment. Hence, this
of prision mayor, as minimum, to twelve (12) years prompted the officers to ask for his name and since
and ten (10) months of reclusion temporal, as he answered that he was Petrus Yau, a British
maximum, meted out against Susana, an national, they asked him for his drivers license and
accomplice, to be proper. car registration but appellant was not able to
produce any. Since he could not produce any drivers
The entire amount of the civil liabilities should be license and car registration, they were supposed to
apportioned among all those who cooperated in the bring him to the police station for investigation,
commission of the crime according to the degrees of their however, when shown a picture of private
liability, respective responsibilities and actual participation. complainant and asked if he knew him, he answered
Accordingly, Petrus should shoulder a greater share in the that the man is being kept in his house. He was
total amount of damages than Susana who was adjudged immediately informed that he was being placed
only as an accomplice. (People v. Montesclaros) under arrest for kidnapping private complainant
Alastair Onglingswam after being informed of his Doctrine: The commission of a felony under Art. 3 of the
constitutional rights. rescue operations of the victim Revised Penal Code requires that an act a punishable act or
proceeded thereafter; omission must be committed, and that it must be committed
with deceit and/or fault.
4. When the police proceeded inside the house they
FACTS:
found a man sitting on the floor chained and
The appellant is appealing to the court regarding his
handcuffed. The man later identified himself as participation in the killing of a certain Loly Penacerrada. He
Alastair Onglingswam; claims that he did not participate in the killing based on the
claim that he was not present in the said act.
5. RTC finds them guilty beyond reasonable doubt and The antecedent facts are as follows:
rejected the twin defenses of alibi and frame-up At around 9:00 p.m. of February 21, 1981, Bartolome
submitted by Petrus and Susana because the same Paja, barangay captain of Brgy. Tipacla, Ajuy, Iloilo,
were unsubstantiated by clear and convincing was awakened by two of the accused. Paja learns that
evidence Fausta killed their landlord, Lloyd Penacerrada, and
would like to surrender to authorities. Knife used in
killing was seen, and blood was found smeared on
6. The CA affirmed the conviction of Petrus and
Faustas dress.
Susana, Hence this assignment of errors.
-Paja immediately ordered a nephew to take spouses
to the police at the Municipal Hall in Poblacon, Ajay,
where the couple informed the police on duty of the
incident.
ISSUES -Several patrolmen, along with Paja and Augusto
proceeded to the residence at Sitio Nabitasan where
the killing incident allegedly occurred, and found the
The focus of this case is the degree of responsibility
body of the deceased, clad in underwear, sprawled
of each accused-appellant for the crime of face down inside the bedroom.
kidnapping for ransom. -Group stayed for an hour in which the scene was
inspected, and a rough sketch of the area was made.
-The next day, a patrolman, accompanied by a
photographer, went back to the scene for further
investigations. Fausta was brought back to the police
Case no. 2 station.
People of the Philippines v. Gonzales -The autopsy of the deceased was performed at 11:20
183 SSCRA 309, March 19, 1990 a.m. Report shows the following:
-Sixteen wounds: five fatal as they
penetrated the internal organs
-Multiple puncture, stab, incision, -Testified on July 27, 1982; at 5 pm on Feb.
and lacerated wounds 21, 1981, he left his work at Brgy. Central,
-The day after the autopsy, Augusto appeared before and walked home, taking a short-cut.
the sub-station and voluntarily surrendered to Police -While passing at the vicinity of the
Corporal Sazon for detention and protective custody Gonzales spouses home at around 8:00 pm,
for having been involved in the killing of the he heard cries for help. Curiosity prompted
deceased. Augusto requests to be taken to where him to approach the place where the shouts
Fausta was already detained. were from.
Based on the investigations conducted, an -15-20 m away from the scene, he hid
information for murder dated August 26, 1981, was filed by himself behind a clump of banana trees, and
the Provincial Fiscal of Iloilo against the spouses. However, saw all the accused ganging upon the
they pleaded not guilty. Before the trial, however, a certain deceased near a threshing platform. He said
Jose Huntoria presented himself to the wife of the deceased. he clearly recognized all the accused as the
Huntoria claims to be a witness of the killing, and on October place was awash in moonlight.
6, 1981, volunteers as a witness for the prosecution. A -After stabbing and hacking the victim, the
reinvestigation of the case was called, in which several more accused lifted his body and carried it to the
were filed as accused, including the appellant. All the house. Huntoria then left home. Upon
accused except for Lenida pleaded not guilty reaching his house, he related what he saw
At the trial, the prosecution presented Dr. Jesus to his wife and mother before going to sleep.
Rojas, the physician who conducted the autopsy on the body, -Eight months after the incident, bothered
Paja, the patrolmen and constabulary members who joined by his conscience and the fact that his father
in the investigation, the widow, and Huntoria. was a tenant of the deceased, he thought of
Dr. Rojas testified that he performed the autopsy at helping the widow. Out of his own volition,
around 11:20 a.m. on Feb. 1981 after the deceased was taken he travelled to the widows house, and
to the municipal hall. He found 4 puncture wounds, 7 stab related to her what he saw
wounds, 4 incisions, and1 laceration; five of these were fatal Except Fausta who admitted killing the deceased as
wounds. Rojas admitted one of two possibilities:- he was trying to rape her, the rest denied participation in the
-Only one weapon might have caused all the crime. The appellant claimed that he was asleep in his house
wounds- which was one kilometer away from the scene of the crime,
-Multiple instruments were used due to the and he knew of the crime only when his grandchildren went
number and different characteristics to his house that night
The trial court disregarded the version of the
The substance of the prosecutions case rested on defense; it believed the prosecutions version.
Huntorias alleged eyewitness account of On appeal to the Court of Appeals, the appellant
theincident, which was as follows: contended that the trial court erred in convicting him on the
basis of the testimony of the lone witness, and in not
appreciating his defense of alibi. The Court found no merit in
the errors, and rejected defense of alibi. Worsening this is -There is nothing in the findings or the evidence that
that the appellate court found the sentence erroneous, and establishes the criminal liability of the appellant as a
upgraded the penalty to that of murder reclusion principal for direct participation under Art. 17, para. 1 of the
temporal/death. The case is now brought upon certification Revised PenalCode.
by the Court of Appeals, hence the appeal -Furthermore, there is nothing in the findings or
ISSUE: evidence that inculpates him by inducement, under
Whether or not the client, under the evidence presented, has paragraph 2 of the same article. Based on the definition of
committed the felony of murder. felonies in Art. 3 of the Revised Penal Code, the prosecutions
evidence could not establish intent nor fault. Recall that the
HELD: elements of felonies include:
NO. Courts analysis of the evidence: -An act or omission
-Investigation conducted left much to be desired. -Act or omission must be punishable
Centeno gave the date of commission as March 21, 1981. The -Act is performed or omission incurred by deceit or
sketch made was troubling, as it did not effectively indicate fault
the extent of the blood stains in the scenes of crime. This -The lone witness could not properly establish any
would have added a lot of weight to any one of the versions acts or omissions done by the appellant. He stated that he
of the incident does not know who hacked or stabbed the victim, thus
-Sazon, who claimed that Gonzales surrendered to implying that he does not know what the appellant did. With
him, failed to state clearly the reason for the surrender. It this, the essential elements of felonies may not even be
may even be possible that Augusto surrendered just so he present.
could be safe from the victims kin. Sazon also admitted that -Furthermore, the fact that there were five stab
Augusto never mentioned to him the participation of other wounds and six accused would imply tha one of them may
persons in the killing. not have caused a grave wound (especially given the
-Rojas statement showed two possibilities for the statement of the physician). This may have been the
killing. Faustas admission that she was the only killer is appellant, and given that there is no evidence that the
plausible. Furthermore, there were only five fatal wounds, appellant caused any of the wounds, coupled with the
which will be discussed later. prosecutions failure to prove the presence of conspiracy
-Huntorias testimony, of which the prosecutions (that is, how many people actually took part in the killing), it
argument solely rests, needs to be examined further. weakens the arguments against the appellant. On the lone
Huntorias claims in his testimony did not exactly match witness:-
with those from his cross-examination. He first claimed that
he recognized the people involved. However, in the cross- Huntorias credibility as a witness is tarnished by
examination, he only saw flashes. This implies that he may two points:
not have recognized anyone at all. 1. He came out eight months after the killing. He
claims that he feared for his life, but there was
On the criminal liability of the appellant: no proof that he was being threatened, nor was
the length of time reasonable given the requires the individual at all times to govern his conduct
circumstances. varies with the nature of the situation in which he is to
2. He is not exactly a disinterested/neutral witness. perform. In a matter so important to the good order of
He admitted to being a tenant of the deceased, society as that in question, where the consequences of a
and stated that one of the reasons why he mistake are necessarily so serious, nothing less than the
testified was because the victim was his highest degree of diligence will satisfy the standard
landlord.- prescribed by the law.
-Under our socioeconomic set-up, a tenant owes the FACTS:
source of his livelihood from his landlord. As such, they Julian Gonzales and Accused Tomasa Delos Reyes were
would do everything to get the landlords to their favor. married in 1897. After some time, the two separated, with
Posing as a witness would have been a convenient way to do Gonzales living elsewhere while Delos Reyes stayed at their
this, especially as he ceased to be employed as early as May home where Gonzales mother also lived. On July 12,1900,
1981. Delos Reyes remarried to Ramon Martinez. Thus, Gonzales
Finally, based on Philippine customs and traditions, filed for a case of bigamy against his wife.
it is unlikely for the appellant to be in the scene of the crime, As her defense, she testified that she honestly believed that
as under our family culture, aging parents are usually her husband was dead when she married Martinez. That
sheltered and insulated from possible harm. It is improbable sometime after they separated, the mother of Gonzales
for the accused to bring their aging father when they were informed her that her son was dead and she believed this to
clearly in better shape than he was, and it was unlikely for be true.
the appellant to offer his services as they were more or less As witness, Gonzales testified that after he left their house,
enough to handle what could have been a perceived enemy. he would still visit his relatives who were left living in their
Although alibi is a weak defense, in cases like this house almost every day until a few days before the trial of
where the participation of the appellant is not clear, it may this case.
be considered. In light of the evidence on record, it may be ISSUE:
sufficient for an acquittal. Was acquittal of the accused for the crime of bigamy proper?
Decision of the CA is reversed and set aside. YES
Appellant acquitted RULING:
Acquittal for bigamy was proper, however, accused should be
(case No. 3 is in pdf format) convicted under Art. 568 (Old Penal Code: reckless
CASE No. 4 imprudence)
U.S. vs. DELOS REYES
G.R. No. 504 September 16, 1902 U.S. vs. Ah Chong (15 Phil. 488)
Doctrine/Topic: Art. 3; Mental element (Mens rea) -
DOCTRINE: Deliberate intent (Dolo) - Mistake of fact
There can be no conviction under the Penal Code where by Had the facts been as Ah Chong believed them to be, he
reason of a mistake of fact the intention to commit the crime would have been justified in killing the intrude. Par. 1, Art XI
does not exist. However, the diligence with which the law
of the Revised Penal Code (RPC) provides that in order for Doctrines and Decision:
the act to be justified, the requisites must be present. The basic principle in our criminal law is that a person is
Requisites (Honest Mistake of Fact): criminally liable for a felony committed by him. Under the
1) Unlawful agression on the part of the victim classical theory on which our penal code is mainly based, the
2) Reasonable necessity of the means employed to basis of criminal liability is human free will. Man is
prevent or repel the unlawful aggression essentially a moral creature with an absolutely free will to
3) Lack of sufficient provocation on the part of the choose between good and evil. When he commits a felonious
person defending himself or criminal act (delito doloso), the act is presumed to have
If the victim was really a robber, forcing his way into the been done voluntarily,i.e., with freedom, intelligence and
room of Ah Chong, there would have been unlawful intent. Man, therefore, should be adjudged or held
aggression, there would have been a necessity on the part of accountable for wrongful acts so long as free will appears
Ah Chong to defend himself and/or his home and the knife unimpaired.
would have been a reasonable means to prevent or repel such In the absence of evidence to the contrary, the law presumes
aggression. that every person is of sound mind and that all acts are
The act done by Ah Chong was merely an act done due to voluntary. The moral and legal presumption under our law is
Honest Mistake of Fact. The Court acquits Ah Chong. that freedom and intelligence constitute the normal
FACTS: condition of a person. This presumption, however, may be
The defendant, Ah Chong, was employed as a cook at overthrown by other factors; and one of these is insanity
Officers quarters. On the night, the defendant, who had which exempts the actor from criminal liability.
received for the night, was suddenly awakened by some ART. 12. Circumstances which exempt from criminal
trying to force open the door of the room. He sat up in bed liability.The following are exempt from criminal liability:
and called out twice, Who is there? He heard no answer 1. An imbecile or an insane person, unless the latter
and was convinced by the noise at the door that it was being has acted during a lucid interval.
pushed open by someone bent upon forcing his way into the When the imbecile or an insane person has committed an act
room. The defendant, fearing that the intruder was a robber which the law defines as a felony (delito), the court shall
or a thief, leaped to his feet and called out: If you enter the order his confinement in one of the hospitals or asylums
room, I will kill you. He was struck just above the knee by established for persons thus afflicted, which he shall not be
the edge of the chair and he thought that the blow had been permitted to leave without first obtaining the permission of
inflicted by the person who had forced the door open, whom the same court.
he supposed to be a burglar. Seizing a common kitchen knife An insane person is exempt from criminal liability unless he
which he kept under his pillow, the defendant struck out has acted during a lucid interval. If the court therefore finds
wildly at the intruder who, it afterwards turned out, was his the accused insane when the alleged crime was committed,
roommate. The roommate eventually died. he shall be acquitted but the court shall order his
ISSUE: Whether or not Ah Chong may be held criminally confinement in a hospital or asylum for treatment until he
responsible for murder in the case at bar. may be released without danger. An acquittal of the accused
does not result in his outright release, but rather in a verdict
People v Estrada
which is followed by commitment of the accused to a mental fully understand the charge against him and to plead
institution. intelligently thereto. It is not clear whether accused-
In the eyes of the law, insanity exists when there is a appellant was of such sound mind as to fully understand the
complete deprivation of intelligence in committing the act. charge against him. It is also not certain whether his plea
Mere abnormality of the mental faculties will not exclude was made intelligently. The plea of not guilty was not made
imputability. The accused must be so insane as to be by accused-appellant but by the trial court because of his
incapable of entertaining a criminal intent. He must be refusal to plead.
deprived of reason and act without the least discernment Section 12, Rule 116 of the 1985 Rules on Criminal Procedure
because there is a complete absence of the power to discern speaks of a mental examination. The human mind is an
or a total deprivation of freedom of the will. entity, and understanding it is not purely an intellectual
Sec. 12. Suspension of arraignment.The arraignment shall be process but depends to a large degree upon emotional and
suspended, if at the time thereof: psychological appreciation.Thus, an intelligent
(a) The accused appears to be suffering from an determination of an accuseds capacity for rational
unsound mental condition which effectively renders understanding ought to rest on a deeper and more
him unable to fully understand the charge against comprehensive diagnosis of his mental condition than
him and to plead intelligently thereto. In such case, laymen can make through observation of his overt behavior.
the court shall order his mental examination and, if Once a medical or psychiatric diagnosis is made, then can
necessary, his confinement for such purpose. the legal question of incompetency be determined by the trial
(b) x x x. court. By this time, the accuseds abilities may be measured
The arraignment of an accused shall be suspended if at the against the specific demands a trial will make upon him.
time thereof he appears to be suffering from an unsound Facts:
mental condition of such nature as to render him unable to That on or about the 27th day of December 1994 in the City
fully understand the charge against him and to plead of Dagupan, Philippines, the accused, Roberto Estrada, being
intelligently thereto. Under these circumstances, the court then armed with a butchers knife, with intent to kill one
must suspend the proceedings and order the mental Rogelio Mararac with treachery and committed in a holy
examination of the accused, and if confinement be necessary place of worship, did then and there, wilfully, unlawfully and
for examination, order such confinement and examination. If criminally, attack, assault and use personal violence upon
the accused is not in full possession of his mental faculties at the latter by stabbing him, hitting him on vital parts of his
the time he is informed at the arraignment of the nature and body with the said weapon, thereby causing his death shortly
cause of the accusation against him, the process is itself a thereafter due to Cardiorespiratory Arrest, Massive
felo de se, for he can neither comprehend the full import of Intrathoracic Hemorrhage, Stab Wound as per Autopsy
the charge nor can he give an intelligent plea thereto. Report and Certificate of Death. At the arraignment,
The fact that accused-appellant was able to answer the accused-appellants counsel, filed an Urgent Motion to
questions asked by the trial court is not conclusive evidence Suspend Arraignment and to Commit Accused to Psychiatric
that he was competent enough to stand trial and assist in his Ward at Baguio General Hospital. It was alleged that
defense. Section 12, Rule 116 speaks of an unsound mental accused-appellant could not properly and intelligently enter
condition that effectively renders [the accused] unable to a plea because he was suffering from a mental defect; that
before the commission of the crime, he was confined at the correccional as the maximum term, there being no
psychiatric ward of the Baguio General Hospital in Baguio aggravating or mitigating circumstances
City. He prayed for the suspension of his arraignment and
the issuance of an order confining him at the said hospital. The petitioner should now be deemed to have served
The motion was opposed by the City Prosecutor. The trial the maximum period imposable for the crime for which she
court, motu proprio, propounded several questions on was convicted, i.e., selling 5.5 grams of dried marijuana
accused-appellant. Finding that the questions were leaves. Although her penalty of life imprisonment had
understood and answered by him intelligently, the court already become final, the beneficial effects of the amendment
denied the motion that same day. provided under R.A. 7659 should be extended to petitioner.
Issue:
Whether or not insanity of a person can exempt him from FACTS:
criminal liability and postpone the trial for a case against Petitoner Jesusa Cruz, presently confined at the
him. Correctional Institution for Women in Mandaluyong City
Whether the judge through intelligent questions can validly serving the penalty of life imprisonment imposed upon her
determine the mental capacity of a person in determining as a consequence of her conviction which became final and
whether to continue with the trial and with his liability in a executory last March 1, 1993 for violation of Section 4,
criminal case. Article II of R.A 6425, filed, through her counsel, the present
petition for habeas corpus. Petitioner alleges that, as of the
date of filing of her herein petition, she has already served
five and a half years of her life sentence. She argues that the
penalty of life imprisonment imposed by the trial court is
excessive considering that the marijuana allegedly taken
JESUSA CRUZ VS CORRECTIONAL INSTITUTION from her was only 5.5 grams or less than 750 grams.
FOR WOMEN IN MANDALUYONG
G.R. NO 125672 Septmeber 27, 1996 ISSUE: After having served five and a half years of her life
sentence, may petitioner -- who was convicted of selling 5.5
RA 7659, which took effect on December 13, 1993, grams of prohibited drugs, namely, dried marijuana leaves --
partly modified the penalties prescribed by R.A. 6425 be now entitled to the beneficent penalty provisions of R.A.
(Dangerous Drugs Act of 1972), that is, inter alia, where the 7659 and be now released from imprisonment?
quantity of prohibited drugs involved is less than 750 grams,
the penalty is reduced to a range of prision
correccional to reclusion perpetua and that where the Case #7
marijuana is less than 250 grams, the penalty to be imposed People of the Philippines v. Daniel Quijada
shall be prision correccional. Moreover, applying the G.R. Nos. 115008-09. July 24, 1996
Indeterminate Sentence Law, the penalty imposable is Article 14 Aggravating Circumstances
further reduced to any period within arresto mayor, as
minimum term, to the medium period of prision Ruling:
The unequivocal intent of the second paragraph of Section 1 aggravating circumstance in homicide
of P.D. No. 1866 is to respect and preserve homicide or or murder. Under an information
murder as a distinct offense penalized under the Revised charging homicide or murder, the fact
Penal Code and to increase the penalty for illegal possession that the death weapon was an
of firearm where such a firearm is used in killing a person. unlicensed firearm cannot be used to
Its clear language yields no intention of the lawmaker to increase the penalty for the second
repeal or modify, pro tanto, Articles 248 and 249 of the offense of homicide or murder to
Revised Penal Code, in such a way that if an unlicensed death. The essential point is that the
firearm is used in the commission of homicide or murder, unlicensed character or condition of
either of these crimes, as the case may be, would only serve the instrument used in destroying
to aggravate the offense of illegal possession of firearm and human life or committing some other
would not anymore be separately punished. Indeed, the crime, is not included in the inventory
words of the subject provision are palpably clear to exclude of aggravating circumstances set out
any suggestion that either of the crimes of homicide and in Article 14 of the Revised Penal
murder, as crimes mala in se under the Revised Penal Code, Code.
is obliterated as such and reduced as a mere aggravating
circumstance in illegal possession of firearm whenever the In short, there is nothing in P.D. No. 1866 that manifests,
unlicensed firearm is used in killing a person. The only even vaguely, a legislative intent to decriminalize homicide
purpose of the provision is to increase the penalty prescribed or murder if either crime is committed with the use of an
in the first paragraph of Section 1 -- reclusion temporal in its unlicensed firearm, or to convert the offense of illegal
maximum period to reclusion perpetua -- to death, possession of firearm as a qualifying circumstance if the
seemingly because of the accused's manifest arrogant firearm so illegally possessed is used in the commission of
defiance and contempt of the law in using an unlicensed homicide or murder. To charge the lawmaker with that
weapon to kill another, but never, at the same time, to intent is to impute an absurdity that would defeat the clear
absolve the accused from any criminal liability for the death intent to preserve the law on homicide and murder and
of the victim. impose a higher penalty for illegal possession of firearm if
Neither is the second paragraph of Section 1 meant to punish such firearm is used in the commission of homicide or
homicide or murder with death if either crime is committed murder.
with the use of an unlicensed firearm, i.e., to consider such
use merely as a qualifying circumstance and not as an
offense. That could not have been the intention of the Facts:
lawmaker because the term "penalty" in the subject provision On or about the 30th day of December, 1992, in the
is obviously meant to be the penalty for illegal possession of municipality of Dauis, province of Bohol, Philippines, the
firearm and not the penalty for homicide or murder. We accused, with intent to kill and without any justifiable
explicitly stated in Tac-an: motive, with treachery and abuse of superior strength, the
There is no law which renders the use accused being then armed with a .38 cal. revolver, while the
of an unlicensed firearm as an victim was unarmed, suddenly attacked the victim without
giving the latter the opportunity to defend himself, and with AGGRAVATING CIRCUMSTANCE - Evident
evident premeditation, the accused having harbored a Premeditation, Unlawful Aggression, Night time,
grudge against the victim a week prior to the incident of Taking Advantage of Public Position
murder, did then and there willfully, unlawfully and MOTIVE - Needed to prove Circumstantial Evidence
feloniously attack, assault and shoot Diosdado Iroy y Nesnea ABERATTIO ICTUS - Mistake in the Identity
with the use of the said firearm, hitting the latter on his head CONSPIRACY - Acting in concert
and causing serious injuries which resulted to his death; to
the damage and prejudice of the heirs of the deceased. That =================================
the accused carried and had in his possession, custody and Pt. I
control a firearm (hand gun) with ammunition, without first =================================
obtaining the necessary permit or license to possess the said
firearm from competent authorities which firearm was Search warrant was issued for inspecting the house and
carried by the said accused outside of his residence and was premises of Francisco Bello on the ground that the police had
used by him in committing the crime of Murder with probable cause that Bello illegally possessed a garand rifle, a
Diosdado Iroy y Nesnea as the victim. thompson submachinegun and two automatic pistols

Accused-appellant Daniel Quijada now appeals from the 1 of 3 teams were walking on the Mariawa road toward the
decision of the Regional Trial Court (RTC) of Bohol premises, when one PC Romero heard rumbling of a jeep,
convicting him of the two offenses separately charged in two heard gunshots and saw flashes of light from the direction of
informations, viz., murder under Article 248 of the Revised Buenaflor, his team member running towards the same.
Penal Code and illegal possession of firearm in its aggravated
form under P.D. No. 1866, and imposing upon him the On the jeep which passed by the deployed policemen were
penalty of reclusion perpetua for the first crime and an Fr. Felix Cappellan, Mrs. Zenaida Stilianopolous Tiongson,
indeterminate penalty ranging from seventeen years, four her six children and the driver returning from mass, fearing
months, and one day, as minimum, to twenty years and one that there might be "people with bad intentions" or hold-
day, as maximum, for the second crime. uppers, Fr. Capellan told the driver to go faster. Shots were
fired at the jeep.
Issue:
Whether or not the RTC is correct in convicting the accused After the shooting incident, the police pursued the mission
of two separate crimes. searching the premises.

People v. Pinto Jr. =================================


Pt. II
JUSTIFYING CIRCUMSTANCE - In the performance =================================
of a Public Duty Although Bello and his parents, Mr. and Mrs. Anduiza, were
not around, the police searched the area and found a
Japanese Springfield rifle, ammunition of a garand rifle,
ammunition of a carbine, live ammunition for a .38 caliber that Rosalio fought back. However, Inocencia did not notice
pistol and 380 bullets for an automatic pistol. any weapon near Rosalio's body
Thereafter, the Chief of Police declared the search
terminated and the entire searching party left for =================================
headquarters Pt. III
=================================
Team 3 was instrued by a "superior officer" "to remain and
maintain peace and order in (the) vicinity including Buenaflor and Pinto were charged: without any justifiable
Mariawa". While he and Buenaflor were patrolling the area, cause or motive, with intent to kill,
at around midnight, they "chanced upon a house" wherein
Bello and his group were staying. They captured four of with treachery and evident premeditation, by means of a Cal.
Bello's bodyguards and tied them to a pili tree with the torn 45 Thompson Sub-Machine Gun and a US Carbin Inland,
shirt of one of the captives he saw a man with a bolo in his Cal. 30 owned respectively by said accused,
hand running towards him. As the man was menacingly near
him, Pinto shot the man later identified as Rosalio Andes (1) shoot one Rosalio Andes,23y/o. inflicting upon him
when he was at a distance of around three meters gunshot wounds causing upon said Rosalio Andes serious
and mortal wounds which led to his instantaneous death.
Inocencia's husband was about to offer Bello a cup of coffee
when she heard a successive burst of gunfire. Bello, who was (2) one for the murder of Richard Tiongson, 9y/o. sustained
the balcony facing the copra kiln ("agonan") with his back a gunshot wound at the back, Richard was operated at the
towards the pili tree, gradually fell to the floor with his hands hospital but he died suffering a hemorrage.
above his head. Then there was another burst of gunfire.
From the kitchen, Inocencia rushed to the door from where (3) another for the frustrated murder of Maria Theresa
she saw a man holding a long firearm, whom she later Tiongson.
identified as Pinto, near the pili tree which was around eight
meters from where Bello was, and another man, also holding (4) Bello died because of "shock secondary to massive
a gun, crouching near the stairs hemorrhage due to multiple gunshot wounds.

Inocencia was about to rush to Bello. Just then a man, whom =================================
Inocencia identified as Buenaflor, came up the house, ISSUES
pointed a gun at Inocencia and her husband and told them to =================================
lay flat on the floor.
(1) Is the defense of fulfillment of a duty under Article 11(5)
When Francisco Andes went up the house, he told Inocencia of the RPC tenable?
that Rosalio was dead. Inocencia went near the pili tree (2) Is there evident premeditation, treachery, night time,
where Rosalio's body was, knelt down and asked the man public position present as Aggravating Circumstances?
with a long firearm why he killed Rosalio. The man answered (3) Is the defense of unlawful aggression tenable?
(4) Is the defense of mistake in the identity of those who (3) As regards the unlawful aggression of Rosalio Andes
were shot in the jeep tenable? against Pinto, we find that if we are to believe Pinto, we have
to stamp full credibility on his statement alone. Even
================================= Buenaflor admitted that he did not see Rosalio Andes attack
RULING Pinto.
=================================
(4) On the jeep coming towards them was owned by the
(1) Originally set out to perform a legal duty: the service of a Anduizas, the appellants acted obviously in the belief that
search warrant on Bello. In the process, however, appellants Bello was its passenger and posthaste they fired upon it even
abused their authority resulting in unauthorized and without any inquiry as to the identity of its passengers
unlawful moves and consequences. Armed with only a search
warrant and the oral order to apprehend Bello, they went Buenaflor's motive for wanting to do away with Bello has
beyond the ambit of their mission and deprived Bello and been established. Such motive provided a circumstantial
two other persons of their lives. evidence leading to the inference that indeed he fired his gun

The defense has to prove that these two requisites are All these pieces of circumstantial evidence point to no other
present: (a) the offender acted in the performance of a duty inference than that Pinto and Buenaflor fired their guns in
and (b) the injury or offense committed be the necessary defiance of their superior officer's order only "to find the
consequence of the due performance or lawful exercise of whereabouts" of Bello
such duty. In the absence of the second requisite, the
justification becomes an incomplete one thereby converting The fact that the victims were different from the ones the
it into a mitigating circumstance under Articles 13 and 69 of appellants intended to injure cannot save them from
the same Code conviction. Aberratio ictus or mistake in the identity of the
victim carries the same gravity as when the accused zeroes in
The police had obtained a search warrant for illegal on his intended victim. The main reason behind this
possession of firearms against Bello even on Christmas day conclusion is the fact that the accused had acted with such a
which was supposed to be a holiday, no such effort was made disregard for the life of the victim(s) without checking
in securing warrant of arrest for Bello's alleged frustrated carefully the latter's identity as to place himself on the same
killing of Botin legal plane as one who kills another willfully, unlawfully and
feloniously. Neither may the fact that the accused made a
(2) On Killing Bello and Andres: mistake in killing one man instead of another be considered
Evident premeditation has not been proven beyond a mitigating circumstance
reasonable doubt in this case but we find that the appellants
indeed took advantage of their public position in It is not even necessary to pinpoint who between Pinto and
perpetrating the crime. Buenaflor actually caused the death of Richard or the
wounding of Maria Theresa in the presence of proof beyond
reasonable doubt that they acted in conspiracy with each On trial, the testimonies of the defendants were contrary to
other. each other but such was corroborated by Irene's testimony
that the defendants fired at Tecson while he was lying in bed.
The lower court is AFFIRMED with MODIFICATIONS that
appellants shall solidarily be liable for each of the three The RTC charged the defendants of Homicide through
murders they committed and, for the frustrated murder of reckless impudence and senteced each an indeterminate
Maria Theresa Tiongson penatly and a fine to pay the heirs of Tecson.

Defendants contended that they incur no criminal liability


People vs. Oanis because they believed that the man was Balagtas and they
Art. 11, no. 5 of RPC acted in innocent mistake of fact in the honest performance
Art. 69 of RPC of their official duties.
Murder
Rule 109, Sec. 2, par. 2 Issue: WON they should be held responsible for the death
caused to Tecson?
Facts:
In the afternoon of Dec. 24, 1938, The Constabulary Held: Yes, they are responsible for tge death of Tecson.
Provincial inspector received a telegram stating " escaped
convict Anselmo Balagtas with bailarina and Irene in The SC are of the opinion that the crime committed is
Cabanatuan get him dead or alive". The Inspector then called murder though specially mitigated. The contention of the
his men, Included is Alberto Galanta, and instructed to defendants that they are not liable because of their defense of
arrest Balagtas, and if overpowered, to follow the instruction "mistake of fact" is not applicable.
contained in the telegram. The same instructions were given
to Chief of Polic Oanis, by the Inspector. In the case of Ah Chong, it was shown that there was an
innocent mistake of fact committed without any fault or
The inspector divided the party into 2 groups to locate carelessness because the accused, having no time or
Balagtas. Galanta and Oanis were group together and opportunity to make a further inquiry, and being pressed by
proceeded to Irene's house. Upon reaching the house, Oanis circumstances to act immediately, had no alternative but to
approached brigida Mallare and asked Irenes location. take the facts as they then appeared to him.

Defendants then went to the room of Irene and upon seeing In the instant case, appellants found no circumstances
a man sleeping with his back towards the door where they whatsorever which would press them to immediate action.
were were, simultaneously or successively fired at him with The person in the room being asleep, appellants had ample
their revolvers. It turned out that the person shot and killed time and opportunity to ascertain his identity without hazard
was not the notorious Balagtas, but an innocent Serapio to themselves, and could even effect a bloodless arrest if any
Tecson, who was Irene's paramour. reasonable effort to that end had been made.
The SC also held that the crime committed by appellants is together with the accessory penalties provided by the law,
not merely criminal negligence, the killing being intentional and to pay the costs
and not accidental. Stating that "a deloberate intent to di an
unlawful act is essentially inconsistent with the idea of Art. 4(2). CRIMINAL RESPONSIBILITY. Criminal
reckless imprudence. Responsibility shall be incurred:
xxx xxx xxx
There is, however, an incomplete justifying circumstance, 2. By any person performing an act which would be an
falling under Art. 11 no. 5 of the RPC. Only one requisite was offense against persons or property, were it not for the
present as the "offenders acted in the performance of a duty inherent impossibility of its accomplishment or on account
or in the lawful exercise of a right". Such being the case, and of the employment of inadequate or ineffectual means.
applying Art 69 od the RPC, the penalty lower by one or two Petitioner contends that, Palangpangan's absence from her
degress than that prescribed by law shall be imposed. room on the night he and his companions riddled it with
bullets made the crime inherently impossible.
Intod vs. Court of Appeals The Revised Penal Code, inspired by the Positivist School,
recognizes in the offender his formidability to punish
FACTS: criminal tendencies in Art. 4(2)
February 4, 1979: Sulpicio Intod, Jorge Pangasian, Santos Legal impossibility occurs where the intended acts, even if
Tubio and Avelino Daligdig went to Salvador Mandaya's completed, would not amount to a crime
house and asked him to go with them to the house of Legal impossibility would apply to those circumstances
Bernardina Palangpangan. Thereafter, they had a meeting where
with Aniceto Dumalagan who told Mandaya that he wanted 1. the motive, desire and expectation is to perform an act in
Palangpangan to be killed because of a land dispute between violation of the law
them and that Mandaya should accompany them. 2. there is intention to perform the physical act
Otherwise, he would also be killed. 3. there is a performance of the intended physical act
February 4, 1979 10:00 pm: All of them armed arrived at 4. the consequence resulting from the intended act does
Palangpangan's house and fired at Palangpangan's bedroom not amount to a crime
but there was no one in the room. o Ex: The impossibility of killing a person already dead
RTC: convicted Intod of attempted murder based on the Factual impossibility occurs when extraneous
testimony of the witness circumstances unknown to the actor or beyond his control
prevent the consummation of the intended crime this case
ISSUE: W/N Intod is guilty attempted murder since it is an o Ex: man who puts his hand in the coat pocket of another
impossible crime under Art. 4 (2) with the intention to steal the latter's wallet and finds the
pocket empty
HELD: YES. petition is hereby GRANTED, the decision of United States: where the offense sought to be committed
respondent Court of Appeals holding Petitioner guilty of is factually impossible or accomplishment - attempt to
Attempted Murder is hereby MODIFIED. sentences him to commit a crime; legally impossible of accomplishment -
suffer the penalty of six (6) months of arresto mayor, cannot be held liable for any crime
5. During trial, the prosecution presented the testimonies of
Case # 12 Mendol, Joseph Velasco (Velasco) and Iluminado
ESCAMILLA V. PEOPLE Garcelazo(Garcelazo), who all positively identified the
G.R. No. 188551 February 27, 2013 petitioner as the shooter of Mendol.
HOMICIDE RTC: found petitioner guilty of frustrated homicide.
CA: affirmed the conviction
DOCTRINE: The intent to kill, as an essential element of Issue: Whether or not the CA is correct in affirming the
homicide at whatever stage, may be before or simultaneous conviction of frustrated homicide?
with the infliction of injuries. The evidence to prove intent to
kill may consist of, inter alia, the means used; the nature, 13. People vs Campuhan [G.R. No. 129433. March
30, 2000]
location and number of wounds sustained by the victim; and
Ruling: NO. MODIFIED. guilty of ATTEMPTED RAPE and
the conduct of the malefactors before, at the time of, or sentenced to an indeterminate prison term of eight (8) years
immediately after the killing of the victim.l1 four (4) months and ten (10) days of prision mayor medium
as minimum, to fourteen (14) years ten (10) months and
Petitioners intent to kill was simultaneous with the infliction twenty (20) days of reclusion temporal medium as
of injuries. Using a gun, he shot the victim in the chest, maximum. Costs de oficio.
petitioner continued to shoot at him three more times, albeit
unsuccessfully. While running, the victim saw his nephew in People v. De la Pea: labia majora must be entered for
front of the house and asked for help. The victim was rape to be consummated
immediately brought to the hospital. The doctor said that the Primo's kneeling position rendered an unbridled
victim would have died if the latter were not brought observation impossible
immediately to the hospital. All these facts belie the absence Crysthel made a categorical statement denying
penetration but her vocabulary is yet as underdeveloped
of petitioners intent to kill the victim.
Corazon narrated that Primo had to hold his penis with
his right hand, thus showing that he had yet to attain an
FACTS: erection to be able to penetrate his victim
1. Around 2am of 01 August 1999, a brawl ensued at the the possibility of Primo's penis having breached Crysthel's
comer of Estrada and Arellano Streets, Manila. Mendol was vagina is belied by the child's own assertion that she resisted
about to ride his tricycle at this intersection while facing Primo's advances by putting her legs close together and that
Arellano Street. she did not feel any intense pain but just felt "not happy"
2. Petitioner, who was standing in front of his store, 30 about what Primo did to her. Thus, she only shouted "Ayo'ko,
meters away from Mendol, shot the later four times, hitting ayo'ko!" not "Aray ko, aray ko!
him once in the upper right portion of his chest. no medical basis to hold that there was sexual contact
3. The victim was brought to the Ospital ng Makati for between the accused and the victim.
treatment and survived because of timely medical attention. FACTS:
4. Assistant City Prosecutor of Manila Fled an Information April 25, 1996 4 pm: Ma. Corazon P. Pamintuan, mother
charging petitioner with frustrated homicide.
of 4-year old Crysthel Pamintuan, went to the ground floor of MANCING (appeal withdrawn res. of 7/10/67),
their house to prepare Milo chocolate drinks for her 2 defendants-appellants. G.R. No. L-21860 February 28,
children. There she met Primo Campuhan, helper of 1974
Conrado Plata Jr., brother of Corazon, who was then busy
filling small plastic bags with water to be frozen into ice in RULING: Although inherently weak and easily fabricated,
the freezer located at the second floor.
the evidence presented by an accused in support of that
Then she heard Crysthel cry, "Ayo'ko, ayo'ko!" so she
defense must be scrutinized with the same care that evidence
went upstairs and saw Primo Campuhan inside her
children's room kneeling before Crysthel whose pajamas or supporting other defenses deserves. When an accused puts
"jogging pants" and panty were already removed, while his up the defense of alibi, the court should not at once have a
short pants were down to his knees and his hands holding mental prejudice against him. For, taken in the light of all
his penis with his right hand the evidence on record, it may be sufficient to acquit him, as
Horrified, she cursed "P - t - ng ina mo, anak ko iyan!" in the case of appellant Inoferio. He should be acquitted
and boxed him several times. He evaded her blows and upon the ground that although his defense, in the nature of
pulled up his pants. He pushed Corazon aside who she tried an alibi, is inherently a weak defense, it should be considered
to block his path. Corazon then ran out and shouted for help sufficient as in this case, to tilt the scale of justice in favor of
thus prompting Vicente, her brother, a cousin and an uncle the accused because the evidence for the prosecution is itself
who were living within their compound, to chase the weak and unconvincing and, therefore, by and large,
Campuhan who was apprehended. They called the barangay insufficient to prove the guilt of the accused beyond
officials who detained. reasonable doubt.
Physical examination yielded negative results as Crysthel
s hymen was intact
Campuhan: Crysthel was in a playing mood and wanted to FACTS: After closing his shop for the day, Chinese
ride on his back when she suddenly pulled him down causing merchant Benito Ching left his store in the public market of
both of them to fall down on the floor. Caloocan. He brought with him a paper bag where all the
RTC: guilty of statutory rape, sentenced him to the proceeds of his sale for the day are placed. While walking
extreme penalty of death with his 2 companions, Libantino and Galvez, 4 men
Thus, subject to automatic review accosted them. Two of the men held Ching and Galvez while
Labantino continued to walk in front of them, initially
ISSUE: W/N it was a consummated statutory rape unaware of the commotion. A .45 caliber pistol as pointed at
Ching while he was held, while another man snatched the
paper bag from him. When Ching tried to call Labantinos
name for help, the gun was shot resulting to a gun wound on
PEOPLE OF THE PHILIPPINES, plaintiff-appellee, vs. Ching. Labantino fled, while Galvez finally freed himself
VIOLETO VILLACORTE, alias BONGING, et al., from the men holding him. The men eventually fled too.
defendants. CRISANTO INOFERIO Y ALINDAO After falling from the gun shot, Ching was able to walk to
alias SANTE, and MARCIANO YUSAY alias
home. His common-law wife brought him to the hospital. He to previous agreement to commit a crime, it being sufficient
died a day after. that the malefactors committed shall have acted in concert
pursuant to the same objective." Hence, conspiracy is proved
Galvez and Labantino both identified Villacorte as one of the if there is convincing evidence to sustain a finding that the
men who accosted them. Upon interrogation, Villacorte malefactors committed an offense in furtherance of a
common objective pursued in concert.
admitted to the crime and identified his companions as
Liability of conspirators. A time-honored rule in
"Roque", "Sante" and "Fred". the corpus of our jurisprudence is that once conspiracy is
proved, all of the conspirators who acted in furtherance of
Consequently, an information for robbery with homicide was the common design are liable as co-principals. This rule of
filed against Villacorte, Roque Guerrero (Roque), Alfredo collective criminal liability emanates from the ensnaring
Handig (Fred) and an alias Sante whose identity was not nature of conspiracy. The concerted action of the
yet ascertained. The information was eventually amended to conspirators in consummating their common purpose is a
implicate Crisanto Inoferio. The trial court acquitted Handig patent display of their evil partnership, and for the
and found Villacorte and Inoferio guilty. Guerrero was consequences of such criminal enterprise they must be held
discharged and used as a state witness. solidarity liable.
Imposition of multiple penalties where
conspirators commit more than one offense. Since in
Only Inoferio pursued this appeal. Inoferio testified that he
conspiracy, the act of one is the act of all, then, perforce, each
first met Villacorte and Handig while in Police custody for of the conspirators is liable for all of the crimes committed in
the investigation. He claims that he knew Guerrero as they furtherance of the conspiracy. Consequently, if the
played cara y cruz together. conspirators commit three separate and distinct crimes of
murder in effecting their common design and purpose, each
ISSUE: Whether or not Inoferios weak alibi could warrant a of them is guilty of three murders and shall suffer the
conviction. corresponding penalty for each offense.
Legality and practicality of imposing multiple
death penalties upon conspirators. An accused who
THE PEOPLE OF THE PHILIPPINES vs. AMADEO was charged with three distinct crimes of murder in a single
PERALTA, ET AL. information was sentenced to two death penalties for two
Doctrine: A conspiracy exists when two or more persons murders, and another accused to thirteen (13) separate death
come to an agreement concerning the commission of a felony penalties for the 13 killings he perpetrated. Therefore there
and decide to commit it. Generally, conspiracy is not a crime appears to be no legal reason why conspirators may not be
except when the law specifically provides a penalty therefore sentenced to multiple death penalties corresponding to the
as in treason, rebellion and sedition. nature and number of crimes they commit in furtherance of
Proof of conspiracy. Conspiracy presupposes the a conspiracy
existence of a preconceived plan or agreement; however, to Ruling. The evidence on record proves beyond
establish conspiracy, "it is not essential that there be proof as peradventure that the accused acted in concert from the
moment they bolted their common brigade, up until the time PEOPLE OF THE PHILIPPINES, plaintiff-appellee,
they killed their last victim, Santos Cruz. While it is true that vs. EDWIN DE VERA yGARCIA, RODERICK GARCIA
Parumog, Larita and Luna did not participate in the actual y GALAMGAM, KENNETH FLORENDO and ELMER
killing of Carriego, nonetheless, as co-conspirators they are CASTRO, accused, EDWIN DE VERA yGARCIA,
equally guilty and collectively liable for in conspiracy the act appellant.
of one is the act of all. It is not indispensable that a co-
conspirator should take a direct part in every act and should Doctrines:
know the part which the others have to perform. Conspiracy
is the common design to commit a felony; it is not Conspiracy - It is axiomatic that the prosecution must
participation in all the details of the execution of the crime. establish conspiracy beyond reasonable doubt. Mere
All those who in one way or another help and cooperate in presence does not amount to conspiracy criminal
the consummation of a felony previously planned are co- conspiracy must be founded on facts, not on mere surmises
principals. Hence, all of the six accused are guilty of the or conjectures.
slaughter of Carriego, Barbosa and Santos Cruz each is
guilty of three separate and distinct crimes of murder. Requisites of Conspiracy:
Facts: On February 16, 1958, while the inmates of the To prove conspiracy, the prosecution must establish the
penitentiary were preparing to attend Sunday mass, that a following three requisites:
fight between two rival members of the "Sigue-Sigue" and 1 that two or more persons came to an agreement;
"OXO" gangs occurred in the plaza where the prisoners were 2 that the agreement concerned the commission of a
assembled, causing a big commotion. The fight was, crime; and
however, quelled, and those involved were led away for 3 that the execution of the felony [was] decided upon.
investigation, while the rest of the prisoners were ordered to
return to their respective quarters. Moments later, another Accomplice, Defined; Elements.
riot erupted in Bldg. 4, as the inmates of brigade 4-A, mostly The Court has held that an accomplice is one who knows the
"OXO" members and sympathizers, destroyed the lock of criminal design of the principal and cooperates knowingly or
their door and then rampaged from one brigade to another. intentionally therewith by an act which, even if not rendered,
The invading OXO clubbed and stabbed to death Jose the crime would be committed just the same.
Carriego, an inmate of 4-B. Afterwards, they forcibly opened
the door of 4-C and killed two more inmates, namely, To hold a person liable as an accomplice, two elements must
Eugenio Barbosa and Santos Cruz. The three victims be present:
sustained injuries which swiftly resulted in their death 1 the community of criminal design; that is, knowing
before they could be brought to the hospital. the criminal design of the principal by direct
participation, he concurs with the latter in his
Issue: Whether or not the commission of the purpose; and
murders was attended with conspiracy? 2 the performance of previous or simultaneous acts
that are not indispensable to the commission of the
crime.
The accused with evident premeditation, treachery and use
Conspirator and Accomplice, Distinguished. of superior strength, attack, assault and employ personal
1 Once conspiracy is proven, the liability is collective violence upon the person of one FREDERICK CAPULONG y
and not individual. The act of one of them is deemed DIZON, hitting him between his eyes and striking him with
the act of all. In the case of an accomplice, the the use of a baseball bat in the mouth, thereby inflicting
liability is one degree lower than that of a principal. upon him serious and mortal wounds. The trial court ruled
2 Conspirators and accomplices have one thing in that there was conspiracy and sentenced all of the accused to
common: they know and agree with the criminal suffer reclusion perpetua mainly on the testimony of an
design. Conspirators, however, know the criminal eyewitness. Specifically, it based its conclusions on the
intention because they themselves have decided following facts: appellant was seen with the other accused
upon such course of action. Accomplices come to inside the victims car; the victim was clearly struck with a
know about it after the principals have reached the blunt object while inside the car, and it was unlikely for
decision, and only then do they agree to cooperate in Florendo to have done it all by himself; moreover, it was
its execution. impossible for De Vera and Garcia to have been unaware of
3 Conspirators decide that a crime should be Florendos dark design on Roderick.
committed; accomplices merely concur in it.
Accomplices do not decide whether the crime should Such witness, Cacao, testified that he saw Appellant De Vera
be committed; they merely assent to the plan and in the car, where an altercation later occurred. Thereafter, he
cooperate in its accomplishment. saw Florendo drag out of the vehicle an apparently disabled
4 Conspirators are the authors of a crime; accomplices Capulong and shoot the victim in the head moments later.
are merely their instruments who perform acts not But, Cacaos testimony contains nothing that could inculpate
essential to the perpetration of the offense. appellant De Vera aside from the fact that he was inside the
car, no other act was imputed to him.
Accomplices; Penalties;
The penalty of an accomplice is one degree lower than that Issues:
of a principal. 1 WON the trial court erred in deciding that there is
conspiracy?
Murder; Aggravating Circumstances; Treachery; 2 WON the trial court erred in aside from treachery,
Abuse of Superior Strength; Treachery absorbs there are other two aggravating circumstances
abuse of superior strength (evident premeditation and abuse of superior
Treachery absorbs abuse of superior strength. The trial court strength).
found that the killing was attended by treachery, evident
premeditation and abuse of superior strength. Hence, there Ruling:
is only one generic aggravating circumstance, not two. 1 YES. The prosecution must establish conspiracy
beyond reasonable doubt. Here, the bare testimony
Facts: of Cacao fails to do so. Cacaos testimony contains
nothing that could inculpate appellant. Aside from
the fact that he was inside the car, no other act was convince the court that the wound sustained would have
imputed to him. Mere presence does not amount to caused the victims death without timely medical
conspiracy. Criminal conspiracy must be founded on attention, the accused should be convicted of attempted
facts, not on mere surmises or conjectures murder and not frustrated murder.
2 YES. Treachery absorbs abuse of superior strength.
Hence, there is only one generic aggravating A treacherous attack is one in which the victim was not
circumstance, not two. afforded any opportunity to defend himself or resist the
attack. The existence of treachery is not solely
determined by the type of weapon used. If it appears that
PEOPLE OF THE PHILIPPINES, Appellee, v. REGIE the weapon was deliberately chosen to insure the
LABAIGA, Appellant. execution of the crime, and to render the victim
G.R. No. 202867, July 15, 2013 defenseless, then treachery may be properly appreciated
against the accused.
ARTICLE PRINCIPLE OF LAW
Article 6 Consummated, frustrated, and attempted FACTS
felonies.
According to the Prosecution, Regie Labiaga shot Gregorio
In Serrano v. People, we distinguished a frustrated Conde outside the latters house. Conde cried for help which
felony from an attempted felony in this manner: prompted his daughters Judy and Glenelyn to rush towards
1 In a frustrated felony, the offender has him. Regie Labiaga shot Judy and told the other accused
performed all the acts of execution which should Balatong Barcenas and Cristy Demapanag that she is
produce the felony as a consequence; whereas in already dead., and they left the vicinity. Judy and Gregorio
an attempted felony, the offender merely were rushed to the hospital wherein Gregorio was treated
commences the commission of a felony directly and fully recovered Judy was declared dead on arrival due to
by overt acts and does not perform all the acts of cardiopulmonary arrest secondary to Cardiac Tamponade
execution. due to gunshot wound.
2 In a frustrated felony, the reason for the non- According to the defense, Regie Labiaga admitted that he
accomplishment of the crime is some cause was present during the shooting incident and claimed that he
independent of the will of the perpetrator; on the acted in self-defense when Gregorio challenged him to a fight
other hand, in an attempted felony, the reason while armed with a shotgun. He alleged that when he tried to
for the non-fulfillment of the crime is a cause or wrest the gun from Gregorio, the shotgun fired without him
accident other than the offenders own knowing whether somebody was hit. Demapanag alleged that
spontaneous desistance. she was 14 km away from the crime scene which was
corroborated by her brother.
In frustrated murder, there must be evidence showing The accused were charged with Murder with the Use of
that the wound would have been fatal were it not for Unlicensed Firearm. The information stated that the
timely medical intervention. If the evidence fails to accused, conspiring, confederating and helping one
another, armed with unlicensed firearm, with deliberate 121 SCRA 389
intent and decided purpose to kill, by means of treachery
and with evident premeditation, did then and there G.R. Nos. L-33466-67
willfully, unlawfully and feloniously attack, assault and
shoot Gregorio Conde with said unlicensed firearm, x x x
thereby performing all the acts of execution which would April 20, 1983
produce the crime of Murder as a consequence, but
nevertheless did not produce it by reason of causes
independent of the will of the accused; that is by the timely
and able medical assistance rendered to said Gregorio
Conde which prevented his death. Intro:
The RTC acquitted Demapanag due to insufficiency of
evidence while Regie Labiaga was convicted of murder and Defense of one's person or rights is treated as a justifying
frustrated murder. CA-Cebu affirmed the conviction.
circumstance under Art. 11, par. 1 of the Revised Penal Code,
but in order for it to be appreciated, the following requisites
ISSUES:
Whether Regie Labiaga is guilty of frustrated or attempted must occur:
murder.
First. Unlawful aggression;
HELD:
In the instant case, it does not appear that the wound Second. Reasonable necessity of the means employed to
sustained by Gregorio Conde was mortal. This was admitted prevent or repel it;
by Dr. Edwin Figura, who examined Gregorio after the
shooting incident.
Third. Lack of sufficient provocation on the part of the
Since Gregorios gunshot wound was not mortal, the SC held
that appellant should be convicted of attempted murder and person defending himself (Art. 11, par. 1, Revised Penal
not frustrated murder. Under Article 51 of the Revised Penal Code, as amended).
Code, the corresponding penalty for attempted murder shall
be two degrees lower than that prescribed for consummated
murder under Article 248, that is, prision correccional in its
maximum period to prision mayor in its medium period. In this case, there was an aggression, not on the person of
appellant, but on his property rights when the victims angrily
ordered the continuance of the fencing. There was also no
Case No. 18
provocation at all since the appellant was asleep before the
commission of the crime. However, the second element is
People vs. Narvaez not present as the killing was disproportionate to the attack.
or incomplete, unless the victim had committed unlawful
aggression against the person who resorted to self-
Facts: defense."30 "Unlawful aggression is defined as an actual
On August 22, 1968, Narvaez shot Fleischer and Rubia as the physical assault, or at least a threat to inflict real imminent
two were constructing a fence that would prevent Narvaez injury, upon a person. In case of threat, it must be offensive
from getting into his house and rice mill. The defendant was and strong, positively showing the wrongful intent to cause
taking a nap when he heard sounds of construction and injury. It presupposes actual, sudden, unexpected or
found fence being made. He tried to stop the group from imminent dangernot merely threatening and intimidating
destroying his house and asked if they could talk things over action. It is present only when the one attacked faces real
but Fleischer responded with "No, gadamit, proceed, go and immediate threat to ones life." 31"Aggression, if not
ahead." Defendant lost his "equilibrium," and shot Fleisher continuous, does not constitute aggression warranting self-
and Rubia with his shotgun. Defendant claims he killed in defense."32
defense of his person and property but the CFI ruled that
Narvaez was guilty and sentenced him to reclusion perpetua, In this case, Flores failed to discharge his burden. Indeed,
to indemnify the heirs, and to pay for moral damages. the nature and number of the gunshot wounds inflicted upon
Jesus further negate the claim of self-defense by the accused.
Records show that Jesus suffered four (4) gunshot wounds in
Issue: the different parts of his body, specifically: on the medial
Whether or not the act of killing of the accused is in defense portion of the left shoulder, between the clavicle and the first
of his person and of his rights, and therefore he should be rib; on the left hypogastric region through the upper right
exempt from criminal liability. quadrant of the abdomen; on the tip of the left buttocks to
the tip of the sacral bone or hip bone; and on the right flank
Flores vs People G.R. No. 181354, February 27, 2013 towards the umbilicus.

Article #/Doctrine: To successfully claim self-defense, the According to Dr. Ruben Escueta, who performed the autopsy
accused must satisfactorily prove the concurrence of the on the victim, the latter died of massive intra-abdominal
elements of self-defense. Under Article 11 of the Revised hemorrhage due to laceration of the liver. 36 If there was any
Penal Code, any person who acts in defense of his truth to Flores claim that he merely acted in self-defense, his
person or rights does not incur any criminal liability first shot on Jesus shoulder, which already caused the latter
provided that the following circumstances concur: to fall on the ground, would have been sufficient to repel the
(1) unlawful aggression; (2) reasonable necessity of attack allegedly initiated by the latter.
the means employed to prevent or repel it; and (3)
lack of sufficient provocation on the part of the But Flores continued shooting Jesus. Considering the
person defending himself. number of gunshot wounds sustained by the victim, the
Court finds it difficult to believe that Flores acted to defend
The most important among all the elements is unlawful himself to preserve his own life. "It has been held in this
aggression. "There can be no self-defense, whether complete regard that the location and presence of several wounds on
the body of the victim provide physical evidence that was on his way back to the house when he saw Flores and his
eloquently refutes allegations of self-defense."37 father talking to each other from a distance of about six (6)
Facts: On August 15, 1989, on the eve of the barangay fiesta meters. Suddenly, Flores shot his father, hitting him on the
in San Roque, Alaminos, Laguna, certain visitors, Ronnie de right shoulder. Flores continued shooting even as Jesus was
Mesa, Noli de Mesa, Marvin Avenido, and Duran, were already lying flat on the ground. Gerry testified that he felt
drinking at the terrace of the house of Jesus. They started hurt to have lost his father.8
drinking at 8:30 oclock in the evening. Jesus, however,
joined his visitors only at around 11:00 oclock after he and Issue: WHETHER THE SANDIGANBAYAN, FIRST
his wife arrived from Sta. Rosa, Laguna, where they tried to DIVISION, GRAVELY ERRED IN NOT GIVING DUE
settle a problem regarding a vehicular accident involving one CREDIT TO PETITIONERS CLAIM OF SELF-DEFENSE.
of their children. The drinking at the terrace was ongoing
when Flores arrived with an M-16 armalite rifle. 4

Duran testified that Jesus stood up from his seat and met
Flores who was heading towards the terrace. After glancing
[G.R. No. 109614-15. March 29, 1996]
at the two, who began talking to each other near the terrace,
Duran focused his attention back to the table. Suddenly, he
heard several gunshots prompting him to duck under the PEOPLE OF THE PHILIPPINES, plaintiff-appellee,
table. Right after the shooting, he looked around and saw the vs. ADRONICO GREGORIO and RICARDO
bloodied body of Jesus lying on the ground. By then, Flores GREGORIO,defendants-appellants.
was no longer in sight.5

Duran immediately helped board Jesus in an owner-type


jeep to be brought to a hospital. Thereafter, Duran, Ronnie
ARTICLE
de Mesa and Noli de Mesa went home. Jesus was brought to
the hospital by his wife and children. Duran did not, at any
ARTICLE 11. JUSTIFYING CIRCUMSTANCES The
time during the occasion, notice the victim carrying a gun
following do not incur any criminal liability:
with him.6
1. Anyone who acts in defense of is person or rights, provided
Gerry narrated that he was going in and out of their house
the following circumstances concur:
before the shooting incident took place, anxiously waiting for
First. Unlawful aggression;
the arrival of his parents from Sta. Rosa, Laguna. His parents
Second. Reasonable necessity of the means
were then attending to his problem regarding a vehicular
employed to prevent or repel it;
accident. When they arrived, Gerry had a short conversation
Third. Lack of sufficient provocation on the part of
with his father, who later joined their visitors at the terrace. 7
the person defending himself.
Gerry was outside their house when he saw Flores across the
street in the company of some members of the CAFGU. He
According to the court the guiding jurisprudential for safekeeping so as to avoid trouble. Complying therewith,
principle has always been that when an accused invokes the Carlos Catorse handed over his samurai, John Villarosa and
justifying circumstance of self-defense, the burden of proof is Remolito Calalas, their respective knives, to Adronico .
shifted to him to prove the elements of that claim; otherwise, Around 1:00 oclock in the morning of May 8, 1986, while the
having admitted the killing, conviction is inescapable. And game of pusoy was still in progress, appellant Ricardo, in a
that he must rely on the strength of his own evidence and not very loud voice, reprimanded Tunggak (son of Adronico)
on the weakness of the prosecution. Having admitted the from peeping at the cards of other players. In response,
killing, appellant has to justify his taking of a life by the Tunggak stood up and also in a very loud voice ordered the
exacting standards of the law. For self-defense to prosper, game stopped).
the following requisites must concur: (1) there must be
unlawful aggression by the victim; (2) that the means Overhearing the incident, Adronico ordered his son
employed to prevent or repel such aggression were downstairs and right there and then scolded and boxed him
reasonable; and (3) that there was lack of sufficient several times. While Adronico was severely beating Tunggak,
provocation on the part of the person defending himself. Carlos Catorse approached and begged Adronico from
further hurting his son so as not to put him to shame before
If the appellants stabbed Carlos Catorse and Marcelo the crowd. Carlos was in this act of pacifying the matter
Lo to defend themselves it certainly defies the reason why between the father and son when suddenly appellant Ricardo
they had to inflict 16 stab wounds and 6 respectively. The stealthily stabbed Carlos from behind with a samurai (the
location, number and gravity of the wounds inflicted on the same samurai deposited by Carlos to Adronico) and
victims belie the appellants contention that they acted in thereafter hacked and stabbed him several times more in
self-defense. The nature and extent of the wounds inflicted different parts of his body: Right after Carlos fell to the
on a victim negate an accused claim of self-defense. ground, Adronico, for his part, repeatedly hacked the victim
with a bolo.

Romeo Catorse, son of Carlos Catorse, ran out of the house.


FACTS Jovito Nicavera also tried to get out of the house but
Adronico hacked him instead with a bolo hitting his left
Around 8:00 PM of May 7, 1986, Carlos Catorse shoulder. Marcelo Lo tried to help his uncle Jovito but
together with his fifteen year old son Romeo Catorse arrived Ricardo, with the same samurai used against Carlos hacked
at the house of appellant Adronico Gregorio at Sitio Bug-as, him on his forearm. Adronico immediately followed and
Barangay Sta. Cruz, Murcia, Negros Occidental, to attend the
using a bolo hacked Marcelo on the nape. Although
wake of the latters grandson.
wounded, Marcelo was able to run out of the house but
Adronico ran after and overtook him. Adronico then hacked
Persons attending the wake were requested by appellant him again.
Adronico to deposit with him any weapon in their possession
Ricardo died during the trial. Adronico on the other hand
interposed self-defense to exculpate himself from criminal
liability. In order to justify self-defense, it is essential that the
attack upon defendant be simultaneous with the
ISSUE killing, or preceded the latter without an appreciable
Whether or not the appellant can validly invoke self- interval of time. (Ferrer, 1 Phil. 56),
defense.

xxx xxx xxx

P/CPL. FAUSTO ANDAL v. SANDIGANBAYAN AND


PEOPLE OF THE PHILIPPINES
The harm caused by one person to another who
G.R. No. L-60159, 06 November 1989 offended or caused him injury, sometime after he
suffered such offense or such injury, does not
constitute an act of self-defense, but an act of
revenge. (Banzuela 31 Phil. 564)

Introduction (Provisions involved, principles,


jurisprudence): In imposing on the appellant the penalty of just one (1) year
of prision correccional, the respondent Court held (which
the Supreme Court affirmed):

The petitioner failed to prove the defense he had raised. The


primordial requisite of self-defense is unlawful aggression.
And for unlawful aggression to be present, there must be a xxx xxx xxx
real danger to life or personal safety. In the instant case,
there was no imminent and real danger to the life or limb of
the petitioner when he shot the deceased, since the latter had
already been disarmed. As former Chief Justice Aquino
Article 69 of the Revised Penal Code vests discretion
states in his book on Criminal Law:
to the court in lowering the penalty either by one or
two degrees whenever incomplete justifying When petitioner Fausto Andal and Quinio went back to the
circumstance exists in a given case like the case at poblacion of Batangas City and parked their vehicle in front
bar. The laudable patience of accused in not of the Philbanking Building at P. Burgos Street, Batangas
retaliating despite repeated insults by a subordinate, City, Macaraig arrived and went straight to the petitioner as
his length of service in the government (since 1957), he was furious and demanded why the petitioner had
and most important, his obsession to inculcate embarrassed him in front of so many people. The petitioner
discipline in his men, to OUR mind, entitle accused denied the charge and Quinio told Macaraig that the
to a two-degree reduction of the penalty prescribed petitioner did not utter defamatory words against him and
by law. Our attitude is a signal to the men in uniform asked him to forget the incident. But still Macaraig
that while WE condemn felonious violence WE challenged the petitioner and fired his gun at the petitioner,
support efforts to maintain discipline in the service. hitting the latter in the middle aspect, lower right knee. The
petitioner was able to wrest the gun from Macaraig when
they grappled for the possession of the gun and two (2)
successive shots were fired at Macaraig by the petitioner
which caused Macaraigs death.

Brief facts:

At about 0700pm on 25 September 1980, Petitioner Fausto Brief issue:


Andal, then a corporal in the Batangas Integrated National
Police, while on patrol aboard a tricycle driven by Police Pfc.
Casiano Quinio, went to the police station at the pier located
at Sta. Clara, Batangas City to check on one of his men, Pfc. Was there an appreciable time lapse between the first
Maximo Macaraig, who was stationed there, because the said aggression, i.e. when deceased shot accused on his knee and
Macaraig had failed to report to police headquarters for the time accused resorted to force by way of firing the two
briefing but Macaraig replied that he did not have to report shots at the deceased? The facts unfolded indicate that there
to police headquarters since he already had his orders and was. This is what happened after accused had grabbed the
told the petitioner: "You report, supsup, ka. gun: (1) He asked deceased, "Why did you fire at me?" (2) He
even turned his head towards his son and instructed him just
to stay in the jeep. (3) His son, Domingo Andal, challenged
deceased to a fight "Sportsman like." (4) Deceased moved
backward 2 meters away from accused. (5) Pfc. Quinio
even thought the trouble was over as he started to get his the part of the victim and, therefore, the plea of self-defense
tricycle. must fail. The narrations of the sequence of events by the
accused, and by the lone alleged eyewitness for the defense,
are unconvincing primarily on account of their inherent
inconsistency and conflict with each other.
Qualifying Circumstances; Under such considerations
Case no. 25 and there being no other evidence to prove that the death of
the victim was the result of meditation, calculation or
reflection, evident premeditation cannot be appreciated to
qualify the killing to murder. The circumstances qualifying
PEOPLE OF THE PHILIPPINES vs. GERARDO or aggravating the act must be proved in an evident and
SAZON, alias "INSIK," incontestable manner. They must be proved as conclusively
as the acts constituting the offense. Thus, for the same
reason, the aggravating circumstance of abuse of superior
G.R. No. 89684 | September 18, 1990 strength cannot be appreciated in this case. Superior
strength may aggravate or qualify a crime, only if it is clearly
REGALADO, J. shown that there was deliberate intent to take advantage of
it. Absence of any evidence to show that the accused
purposely sought to use their superior strength to their
DOCTRINE / RULING: advantage in this case, a finding to that effect by the trial
Self-Defense; Well-entrenched is the rule that where the court cannot be sustained.
accused invokes self-defense, it is incumbent upon him to Conspiracy; The fact that appellant did not inflict the
prove by clear and convincing evidence that he indeed acted mortal wound upon the deceased is of no moment, since the
in defense of himself. He must rely on the strength of his existence of conspiracy was satisfactorily shown by the
own evidence and not on the weakness of the prosecution. evidence. The coordinated acts of appellant and Altejos of
For, even if the prosecution evidence is weak, it could not be immediately following the victim and jointly confronting him
disbelieved after the accused himself had admitted the thereafter reveal a concordance and unity of thought which
killing. resulted in the encounter. The circumstances that after the
accused shot the victim in the forearm and, while he and the
It is a statutory and doctrinal requirement that for the victim were grappling for appellants gun, Altejos stabbed the
justifying circumstance of self-defense, the presence of victim to death, indicate closeness and coordination of their
unlawful aggression is a condition sine qua non. There can action geared towards a common purpose, that is, to kill the
be no self-defense, complete or incomplete, unless the victim victim. Proof of a previous agreement to commit the crime is
has committed an unlawful aggression against the person not absolutely essential to establish a conspiracy. It is
defending himself. In the present case, the burden of sufficient that the accused be shown to have acted in concert
evidence having been shifted, we hold that the defense failed pursuant to the same objective, as such circumstance is
to establish the primary element of unlawful aggression on invariably indicative of a conspiratorial agreement.
The rule is that where a conspiracy is proven, a showing as to Appellants version of the incident, however, differs. He
who inflicted the fatal wound is not required to sustain a admits having shot Longno but pleads self-defense. He
conviction. The act of one in killing the victim becomes the claims that Altejos for help to have a .22 caliber revolver
act of all the accused. repaired and appellant was taking the revolver to a
policeman friend of his. On their way, appellant saw Longno
FACTS: from a distance. Upon his approach, Longno allegedly said,
Ernesto Romualdez was accosted by appellant near the Insik, I heard that you are not afraid of me. Maybe you want
barangay hall for allegedly circulating the rumor that to be taught a lesson.
appellant and his companions were engaged in stealing.
Upon confrontation, appellant boxed Romualdez which Appellant claims that the deceased had a revolver tucked in
caused the latter to fall. Wilfredo Longno, who was then his waist and was about to draw the same. He, therefore,
present at the scene, approached and helped the fallen parried the gun but it fired hitting one of appellants left
Romualdez and pushed appellant away. This apparently fingers which was later amputated. It was then that appellant
angered appellant who, in his native dialect said, Andam ka pulled out his gun and shot Longno in the forearm. Appellant
lang Inday kay patyon ta guid, (Watch out Inday for I will and Longno afterwards grappled for the gun. Altejos
kill you) to which Longno retorted, Just do it. allegedly tried to separate appellant and Longno but he was
brushed aside by the latter. In the course of their struggle,
Two days later, appellant and his cousin, Cornelio Altejos, Altejos then shouted to appellant, I stabbed Inday, run,
were drinking softdrinks at the store of Gloria Aposaga when and so he and Altejos ran away.
Longno passed by. Thereupon, appellant and Altejos left
their softdrinks half-consumed and followed Longno who
eventually reached the bench near the public faucet where a ISSUE/S:
group of guys were sitting and joined the group in their 1 Did the accused act in self-defense?
conversation. Shortly thereafter, appellant and Altejos 2 Whether or not qualifying circumstances are
arrived and appellant accosted and pointed a gun at Longno, present?
saying, Maano ka? (What are you going to do?). Longno 3 Is there a conspiracy between the accused and
then faced appellant and said, Brod, tiruha lang. (Brod, Altejos?
just shoot.)
Note:
Apparently irked by the response, appellant fired the gun, Insofar as Cornelio Altejos is concerned, however, the trial
hitting Longno in the left forearm. Dullete, Canoso and court never acquired jurisdiction over him and he can
Ramos then scampered for safety as appellant and the neither be convicted nor exculpated herein. References in
wounded Longno grappled for the gun. It was while the two this judgment to him are, therefore, obiter and with no
were thus struggling that Altejos stabbed Longno in the binding effect on him.
chest, after which both appellant and Altejos ran away.

PEOPLE v. ALCONGA
Facts: succeeded in showing that Barions aggression was
As the deceased Barion and the defendant Alconga were proportionate to his killing his already defeated
playing black jack, the former became outraged and adversary. Also, it is noteworthy to mention that at
expressed his rage at the latter for his cheating tactics. Two Alconga, as a guard by profession, was the holder of
(2) days later, making good the threat uttered after the game, more deadly weapons and has superior fighting ability
Barion approached the defendant and swung his pingahan, a than Barion.
bamboo stick. A fight ensued afterwards, upon which
Alconga gained the upper hand by firing his revolver at
Barion and eventually inflicting a mortal bolo blow, slashing
the latters cranium. The argument of self defense was raised THE PEOPLE OF THE PHILIPPINES vs. BENJAMIN
by Alconga. GONDAYAO, alias "BEN", ET AL.
Issues:
1 Whether or not the Alcongas plea of self defense can
be sustained Article 11. Justifying Circumstances- 1. Anyone who
2 Whether or not Barions provocation is considered a acts in defense of his person or rights, provided that the
mitigating circumstance following circumstances concur: First. Unlawful aggression;
Ruling: Second. Reasonable necessity of the means employed to
1 NO. prevent or repel it; Third. Lack of sufficient provocation on
There were two (2) stages in the fight- the first wherein the part of the person defending himself.
the deceased was the aggressor and the second wherein
there is no more aggression, since Barion was already Article 13. Mitigating Circumstance- 1. Those
injured and fleeing. Under such circumstances, mentioned in the preceding chapter, when all the requisites
Alcongas plea of self-defense (as a justifying necessary to justify the act or to exempt from criminal
circumstance) during the second stage of the fight liability in the respective cases are not attendant.
cannot be sustained. There can be no defense where
there is no aggression, given that the aggression must be
deemed to have ceased upon the flight of the deceased Article 14. Aggravating Circumstance- 16. That the act
during the first stage. A fleeing man is not dangerous to be committed with treachery (alevosia).
the one from whom he flees.
2 NO. There is treachery when the offender commits any of the
Section 4, Article 13 of the Revised Penal Code crimes against the person, employing means, methods, or
requires that there be sufficient provocation or threat forms in the execution thereof which tend directly and
on the part of the offended party immediately preceding specially to insure its execution, without risk to himself
the act. Sufficiency, as defined by Justice Albert, means arising from the defense which the offended party might
that it should be proportionate to the act committed make.
and adequate to stir one to its commission. It would
seem self-evident that Alconga could never have
No unlawful aggression when there is an agreement to fight. (Benjamin) thrust the dagger, with his left hand, into the
The challenge to a fight must be accepted. The reason for the back of Piol, causing therein another stab wound almost at
rule is that each of the protagonists is at once assailant and right angle with his body, like the first. Just why, being in the
assaulted and neither can invoke the right to self-defense precarious condition he depicted himself, Benjamin chose to
because aggression which is incident in the fight is bound to stab Piol in such an awkward, inconvenient and unbelievable
arise from one or the other of the combatants. manner, the defense has not even tried to explain.
Regardless of the foregoing, an injury inflicted in this fashion
Unlawful aggression must come, directly or indirectly, from on Piol's back, which was allegedly pressed against the
the person who was subsequently attacked by the accused. ground, would have necessarily been much more slanting
than the first, instead of being almost perpendicular to the
RULING: Benjamins story is manifestly artificious and body. The fact of the matter and this has been established
unworthy of credence. It should be noted that, according to by the testimony of the Chief of Police, whose impartiality
Benjamin, he held the dagger, even before they fell from the and veracity are not contested is that Piol was then lying
"papag"; that the dagger was still in his hand when Piol down, not on his back, but on his stomach with Benjamin on
allegedly struck his face with a stone; and that he top of him. This explains why and how he (Benjamin)
(Benjamin), in turn, took another piece of stone, and hit Piol managed to stab Piol on the back. It, likewise, shows that
with it. This would have been impossible, however, unless Piol could not have struck Benjamin on the face with a stone,
Benjamin first released the dagger, which he then held; but, much less squeezed his neck.
We cannot believe that he, or anybody for that matter, would
have done so under the circumstances. Benjamin stabbed Piol twice from behind, after disarming
him. Considering, moreover, that Benjamin had provoked
Again, when Piol allegedly squeezed the neck of Benjamin, the incident, by hurling uncomplimentary remarks at his
the latter threw the stone away and picked up the dagger political opponents, one of whom was Piol; that such
once more. He would thus have Us believe that, in order to remarks led to an altercation with Piol, in consequence of
get the stone with which he claimed to have hit Piol on the which, stones were thrown at him, hitting him on the head;
head, he put the dagger in a convenient place from which, at that when, owing to the impact of said stone, which could
the opportune moment, he got it back to inflict the second have rendered him groggy, and the lacerated injuries thus
stab wound. The context of Benjamin's story does not convey sustained by him, Piol prepared himself to fight by drawing
the idea that he had such a control of the situation as to be out his dagger, Benjamin accepted the challenge resulting
able to choose the place where he would put the dagger and from this act, by "rushing" to his encounter and grappling
the time he would retrieve it. with him; and that, accordingly, Benjamin cannot be given
the benefit of either complete or incomplete self-
defense. Although Piol was stabbed from behind, Benjamin
But, this is not all. Instead of stabbing Piol on the stomach,
did not act with treachery, for this was merely an incident of
for, by this time, he was again lying down on his back
according to the defense with Benjamin on top of him, he
their struggle, which had begun with both contenders facing Benjamin assumed full responsibility for said injuries, which
each other, each prepared for the fight that ensued. he claimed to have inflicted in self-defense.

The crime committed by Benjamin Gondayao is, therefore, Benjamin testified that his remark, about the failure to give a
that of homicide, and no modifying circumstance having blow-out on the part of the winner in the elections, was
attended its perpetration, the penalty therefor (reclusion addressed to his nephew, Rudy Natividad; that, when Piol
temporal) should be imposed in its medium period. resented said remark, Benjamin replied that the same was
Pursuant to the Indeterminate Sentence Law, he should, not aimed at him, and that, perhaps, he (Piol) is a
accordingly, be sentenced to an indeterminate penalty "nacionalista".
ranging from 8 years and 1 day of prison mayor, as
minimum, to 14 years, 8 months and 1 day of reclusion ISSUE: Whether or not the accused acted in self-defense.
temporal as maximum, with the corresponding accessory
penalties.
G.R. No. L-28129 October 31, 1969
FACTS: Appeal, taken by defendants Benjamin Gondayao ELIAS VALCORZA, petitioner,
and Anoy Gondayao from a decision convicting them of the vs.
crime of murder of Orlando Piol, qualified by treachery. PEOPLE OF THE PHILIPPINES, respondent.

RULING
After consuming a bottle of gin, Benjamin, repeatedly cursed The petitioner is acquitted.
the "Nacionalistas" for not offering a drink despite their
victory in the polls; that, as Piol who was nearby, replied by ART. 11. Justifying circumstances. - The following do not
cursing the losers in the elections, an exchange of unfriendly incur any criminal liability
utterances followed; that, eventually, the group headed by Paragraph 5. Any person who acts in the fulfillment
Benjamin picked up stones and threw them at Piol, who was of a duty or in the lawful exercise of a right or office.
hit several times on the head; that, as the same began to
bleed, Piol drew out a dagger and approached Benjamin, The facts and circumstances constrain us to hold that the act
who, forthwith, embraced him; that, grappling with each thus performed by petitioner, which unfortunately resulted
other, they both fell down. in the death of the escaping detainee, was committed in the
performance of his official duty and was more or less
necessary to prevent the escaping prisoner from successfully
After grappling with Benjamin Gondayao, Orlando Piol eluding the officers of the law. To hold him guilty of
appeared to have in addition to several lacerations on the homicide may have the effect of demoralizing police officers
head two (2) stab wounds on the back, 4 to 4-1/2 inches discharging official functions identical or similar to those in
deep, in consequence of which he died due to a massive the performance of which petitioner was engaged at the time
internal hemorrhage. Immediately after the occurrence, he fired at the deceased Pimentel, with the result that
thereafter We would have half-hearted and dispirited efforts justify this finding beyond reasonable doubt. There is no
on their part to comply with such official duty. This of showing that the killing was agreed upon between them
course, would be to the great detriment of public interest. beforehand. No motive for it has been shown other than the
provocation given by the deceased; and such motive was true
FACTS: only insofar as Avelino was concerned. The circumstances
The deceased, Roberto Pimentel was a detention prisoner indicate that if Jose embraced Rodrigo and rendered him
who escaped. While in search for the escaped prisoner, Sgt. helpless, it was to stop him from further hitting Avelino with
Daiton saw a person approaching slowly under the bridge his fists. However, Jose is not entirely free from liability, for
and he ordered him to halt. The latter instead of doing so, it has been established that even after the first knife thrust
jumped down into the creek spanned by the bridge. Roberto had been delivered he did not try to stop Avelino, either by
Pimentel emerged suddenly from the bushes and lunged at word or overt act. Instead Jose continued to hold Rodrigo,
the appellant Valcorza, hitting him with a stone and causing even forced him down on the bamboo bed with Avelino still
him to fall to the ground. Appellant Valcorza regained his pressing the attack. Withal it cannot be said that Jose's
composure and immediately chased the deceased, as the cooperation was such that without it the offense would not
deceased did not heed his order to stop, appellant fired four have been accomplished. But although not indispensable, it
times into the air, and a fifth shot at Pimentel as the latter was a contributing factor. If Jose's initial intent was free
was in an act of again jumping down into another part of the from guilt, it became tainted after he saw the first knife
creek. The members of the patrol team went down into the thrust delivered. The thirteen wounds must have taken an
water to locate Pimentel and they saw him floating, with a appreciable interval of time to inflict, and Jose's cooperation
wound on his back. Elias Valcorza surrendered himself and facilitated their infliction. He must therefore be held liable as
his firearm to the Chief of Police an accomplice.

ISSUE: Facts: Avelino was eating his lunch inside one of the
Whether or not peace officer Valcorza was justified in eateries dotting the market site, when Rodrigo Aringo alias
shooting the deceased Diego, a baggage boy in the same market, approached him
and demanded his fee for having carried Avelino's baggage.
When Avelino said he was willing to pay for the services
PEOPLE OF THE PHILIPPINES vs. AVELINO rendered at noon, but not for those rendered earlier in the
MANANSALA, JR. and JOSE MANANSALA morning, Rodrigo brusquely brushed Avelino's hand aside
G.R. No. L-23514, February 17, 1970 and instantly gave him a fist blow in the face. A quarrel
between them ensued and Jose Manansala, Avelino's uncle,
Article 17 of the RPC; Article 18 of the RPC states that noticed the commotion and so he went there and had
Accomplices are those persons who, not being included in Rodrigo in a tight embrace. While in that position, Avelino
Art.17, cooperate in the execution of the offense by previous stabbed him with a balisong, or Batangas knife thirteen
or simultaneous acts. In this case, Jose Manansala was found times which resulted to Rodrigo's death. Avelino and his
guilty as co-principal on the ground that there was concert of uncle, Jose were charged with murder and Jose was
action between him and his nephew. The evidence does not considered as a co-principal by the CFI.
employed to prevent or repel the attack; and (3) lack of
Issue: Whether or not Jose Manansala was correctly sufficient provocation on the part of the person defending
considered as a co-principal by the CFI? himself.
It was the deceased who drew his gun towards the
Case #32. accused. Not having enough time to think rationally how to
People of the Philippines vs Jose Encomienda deal with the situation, the accuseds act of hacking the left
GR No L-26750; August 18, 1972 arm of deceased is justified due to immediate danger to his
life as the former was about to grab gun from his free left
Article 11, Revised Penal Code/ Legitimate Self- hand. Provocation came from the deceased (not the accused)
Defense, elements: (1) unlawful aggression on part of the as he ordered the accused to vacate land and drew his gun
victim; (2) reasonable necessity of the means, employed to when he did not like the response of the former.
prevent or repel the attack; and (3) lack of sufficient PEOPLE VS. JAURIGUE
provocation on the part of the person defending himself C.A. No. 384, February 21, 1946
Illegal Aggression, what constitutes: Illegal Doctrine:
aggression is equivalent to assault or at least The attempt to rape a woman constitutes an unlawful
threatened assault of immediate and imminent kind. aggression sufficient to put her in a state of legitimate
Reasonable Necessity of the Means defense, inasmuch as a woman's honor cannot but be
Employed, what constitutes: Reasonable esteemed as a right as precious, if not more, than her very
necessity of the means employed does not imply existence; and it is evident that a woman who, thus
material commensurability between the means of imperiled, wounds, nay kills the offender, should be afforded
attack and defense. When the law requires is rational exemption from criminal liability, since such killing cannot
equivalence, in the consideration of which will enter be considered a crime from the moment it became the only
as principal factors the emergency, the imminent means left for her to protect her honor from so great an
danger to which the person attacked is exposed and outrage (1 Viada, Codigo Penal, 5th ed., p. 301; People vs.
the instinct, more than the reason, that moves or Luague and Alcansare, 62 Phil., 504).
impels the defense, and the proportionateness As long as there is actual danger of being raped, a woman is
thereof does not depend upon the harm done, but justified in killing her aggressor in the defense of her honor.
rests upon the imminent danger of such injury. When the deceased sat by the side of the appellant on the
same bench, near the door of the barrio chapel and placed
Facts: Accused was charged with the crime of murder his hand on the upper portion of her right thigh without her
aggravated by recidivism. He interposed the lawful self- consent, the said chapel was lighted with electric lights, and
defense as per Article 11 of the Revised Penal Code. there were already several people inside the chapel,
Issue: Whether or not the accused satisfied the elements of including her own father and the barrio lieutenant and
lawful self-defense? other dignitaries of the organization; and under the
Held: Yes, the accused sufficiently proved the elements for circumstances, there was and there could be no
lawful self-defense. Elements are: (1) unlawful aggression on possibility of her being raped. And when she gave Amado
part of the victim; (2) reasonable necessity of the means, Capina a thrust at his neck, inflicting upon him a mortal
wound and causing his death a few moments later, the Vicente, were prosecuted in the Court of First Instance of
means employed by her in the defense of her honor Ilocos Sur for the crime of homicide. The said accused
was evidently excessive; and under the facts and conspired to attack Ignacio Cambaliza and inflicted a mortal
circumstances of the case, she cannot be legally declared wound upon him which he died in a few minutes afterwards.
completely exempt from criminal liability. Issue:
The law prescribes the penalty of reclusion temporal for the Whether or not the accused Constante can invoke self
crime of homicide; and if it should be reduced by two defense?
degrees, the penalty to be imposed in the instant case is that
of prision correccional; and pursuant to the provisions of Ruling:
section 1 of Act No. 4103 of the Philippine Legislature, The selfdefense invoked by the accused was incomplete, for,
known as the Indeterminate Sentence Law, herein defendant although the appellant was unlawfully attacked by the
and appellant should be sentenced to an indeterminate deceased and compelled to employ reasonable means to
penalty ranging from arresto mayor in its medium degree, to defend himself, he is responsible for provoking the attack.
prision correccional in its medium degree.
PEOPLE vs. HERNANDEZ
Facts: Inside the chapel of the 7th day Adventist GR NO. L-6025, MAY 30 1964
Church, Amado Capina sat beside the appellant and with the Labrador, J.:
greatest of impudence, placed his hand on the upperpart of
her right thigh. On observing this highly improper Article 136. Conspiracy and proposal to commit rebellion or
and offensive conduct, Avelina Jaurigue, conscious of insurrection
her personal dignity and honor, pulled out a fan knife and
stabbed Amado once at the base of the left side of the neck FACTS:
Amado Capina died from the wound a few minutes
later. Appellant Avelina Jaurigue was subsequently tried About March 15, 1945, Amado Hernandez and other
and convicted of the crime of Homicide. appellants were accused of conspiring, confederating and
Issue: Whether or not appellant Jaurige acted in the cooperating with each other, as well as with the thirty-one
legitimate defense of her honor and that she should be (31) defendants charged in the criminal cases of the Court of
completely absolved of all criminal responsibility. First Instance of Manila. They were accused of being
members of PKP Community Party of the Philippines which
People vs Sotelo was actively engaged in an armed rebellion against the
55 PHIL 403 government of the Philippines. With the party of
December 13, 1930 HUKBALAHAP (Hukbo ng Bayan Laban sa mga Hapon),
they committed the crime of rebellion causing murder,
Article 13 of the RPC: Incomplete Self Defense pillage, looting plunder, etc., enumerated in 13 attacks on
government forces or civilians by HUKS.
Facts:
The Sotelo brothers, namely, Constante, Dominador, and
ISSUE: Does his or anyones membership in the communist fostered by the desire to secure the labor vote to support his
party per se render Hernandez or any Communist guilty of political ambitions. It is doubtful whether his desire to foster
conspiracy to commit rebellion under the provisions of the labor union of which he was the head was impelled by an
Article 136 of the RPC? actual desire to advance the cause of Communism, not
merely to advance his political aspirations.
HELD:
Insofar as the appellant's alleged activities as a Communist
No. The advocacy of Communism or Communistic theory are concerned, We have not found, nor has any particular act
and principle is not to be considered as a criminal act of on his part been pointed to Us, which would indicate that he
conspiracy unless transformed or converted into an advocacy had advocated action or the use of force in securing the ends
of action. In the very nature of things, mere advocacy of a of Communism. True it is, he had friends among the leaders
theory or principle is insufficient unless the communist of the Communist Party, and especially the heads of the
advocates action, immediate and positive, the actual rebellion, but this notwithstanding, evidence is wanting to
agreement to start an uprising or rebellion or an agreement show that he ever attended their meetings, or collaborated
forged to use force and violence in an uprising of the working and conspired with said leaders in planning and encouraging
class to overthrow constituted authority and seize the reins the acts of rebellion, or advancing the cause thereof. Insofar
of Government itself. Unless action is actually advocated or as the furnishing of the mimeograph machine and clothes is
intended or contemplated, the Communist is a mere theorist, concerned, it appears that he acted merely as an
merely holding belief in the supremacy of the proletariat a intermediary, who passed said machine and clothes on to
Communist does not yet advocate the seizing of the reins of others. It does not appear that he himself furnished funds or
Government by it. As a theorist the Communist is not yet material help of his own to the members of the rebellion or
actually considered as engaging in the criminal field subject to the forces of the rebellion in the field.
to punishment. Only when the Communist advocates action
and actual uprising, war or otherwise, does he become guilty But the very act or conduct of his in refusing to go
of conspiracy to commit rebellion. underground, in spite of the apparent desire of the chief of
the rebellion, is clear proof of his non-participation in the
The most important activity of appellant Hernandez appears conspiracy to engage in or to foster the rebellion or the
to be the propagation of improvement of conditions of labor uprising.
through his organization, the CLO. While the CLO of which
he is the founder and active president, has communistic
tendencies, its activity refers to the strengthening of the
unity and cooperation between labor elements and preparing G.R. No. L-4445 February 28, 1955
them for struggle; they are not yet indoctrinated in the need THE PEOPLE OF THE PHILIPPINES, plaintiff-
of an actual war with or against Capitalism. The appellant appellee,
was a politician and a labor leader and it is not unreasonable vs.
to suspect that his labor activities especially in connection MANUEL BERONILLA, FILIPINO VELASCO,
with the CLO and other trade unions, were impelled and
POLICARPIO PACULDO, and JACINTO an alleged conspirator, were indicted in the Court of First
ADRIATICO, defendants-appellants Instance of Abra for murder, for allegedly conspiring and
FACTS: confederating in the execution of Arsenio Borjal.
Arsenio Borjal was the elected mayor of La Paz, Abra, at the
outbreak of war, and continued to serve as Mayor during the ISSUE:
Japanese occupation, until March 10, 1943, when he moved whether or not this message, originally sent to Arnold's
to Bangued because of an attempt upon his life by unknown quarters in San Esteban, Ilocos Sur, was relayed by the latter
persons. On December 18, 1944, appellant Manuel Beronilla to appellant Beronilla in La Paz, Abra, on the morning of
was appointed Military Mayor of La Paz by Lt. Col. R. H. April 18, 1945, together with the package of records of
Arnold, regimental commander of the 15th Infantry, Borjal's trial that was admittedly returned to and received by
Philippine Army, operating as a guerrilla unit in the province Beronilla on that date, after review thereof by Arnold
of Abra. (Exhibit 8-8-a).
RULING:
Sometime in March, 1945, while the operations for the
liberation of the province of Abra were in progress, Arsenio We have carefully examined the evidence on this important
Borjal returned to La Paz with his family in order to escape issue, and find no satisfactory proof that Beronilla did
the bombing of Bangued. Beronilla, pursuant to his actually receive the radiogram Exhibit H or any copy thereof.
instructions, placed Borjal under custody and asked the The accused roundly denied it.
residents of La Paz to file complaints against him. In no time,
charges of espionage, aiding the enemy, and abuse of . In addition to Balmaceda was contradicted by Bayken,
authority were filed against Borjal. another prosecution witness, as to the hatching of the alleged
conspiracy to kill Borjal. Balmaceda claimed that the
The trial lasted 19 days up to April 10, 1945; the jury found accused-appellants decided to kill Borjal in the early evening
Borjal guilty on all accounts and imposed upon him of April 18, while Bayken testified that the agreement was
instruction from his superiors. And on the night of the same made about ten o'clock in the morning, shortly after the
day, April 18, 1945, Beronilla ordered the execution of Borjal. accused had denied Borjal's petition to be allowed to hear
Jacinto Adriatico acted as executioner and Antonio Palope as mass.
grave digger. Our conclusion is that Lt. Col. Arnold, for some reason that
can not now be ascertained, failed to transmit the
Two years thereafter, Manuel Beronilla as military mayor, Volckmann message to Beronilla. And this being so, the
Policarpio Paculdo as Clerk of the jury, Felix Alverne and charge of criminal conspiracy to do away with Borjal must be
Juan Balmaceda as prosecutors, Jesus Labuguen, Delfin rejected, because the accused had no need to conspire
Labuguen, Filemon Labuguen, Servillano Afos, Andres Afos, against a man who was, to their knowledge, duly sentenced
Benjamin Adriatico, Juanito Casel, Santiago Casel, Mariano to death.
Ajel, Felix Murphy, Benjamin Abella, and Pedro Turqueza as
members of the jury, Jacinto Adriatico as executioner, decided that the concurrence of personal hatred and
Severo Afos as grave digger, and Father Filipino Velasco as collaboration with the enemy as motives for a liquidation
does not operate to exclude the case from the benefits of the perpetua, conformably to Article 63 of the Revised
Amnesty claimed by appellants, since then "it may not be Penal Code.
held that the manslaughter stemmed from purely personal
motives" Actually, the conduct of the appellants does not FACTS:
dispose that these appellants were impelled by malice. The accused PO3 Ferdinand Fallorina y Fernando, with
The lower Court, after finding that the late Arsenio Borjal intent to kill, by means of treachery and taking advantage of
had really committed treasonable acts, (causing soldiers and superior strength, did then and there, wilfully, unlawfully
civilians to be tortured, and hidden American officers to be and feloniously attack, assault and employ personal violence
captured by the Japanese) expressly declared that "the Court upon the person of VINCENT JOROJORO, JR. y MORADAS,
is convinced that it was not for political or personal reason a minor, eleven (11) years of age, by then and there, shooting
that the accused decided to kill Arsenio Borjal" him with a gun, hitting him on the head, thereby inflicting
upon him serious and mortal wound which was the direct
Actus non facit reum nisi mens si rea. and immediate cause of his death, to the damage and
To constitute a crime, the act must, except in certain prejudice of the heirs of the said offended party.
crimes made such by statute, be accompanied by a The trial court rendered judgment convicting the appellant
criminal intent, or by such negligence or indifference of murder, qualified by treachery and aggravated by abuse of
to duty or to consequence, as, in law, is equivalent to public position. The trial court did not appreciate in favor of
criminal intent. The maxim is, actus non facit reum, the appellant the mitigating circumstance of voluntary
nisi mens rea-a crime is not committed if the minds surrender.
of the person performing the act complained of be ISSUE:
innocent. (U. S. vs. Catolico, 18 Phil., 507). Whether or not the appellant is exempt from criminal
liability.

G.R. No. 137347 March 4, 2004 RULING:

PEOPLE OF THE PHILIPPINES, appellee, The appellant was burdened to prove, with clear and
vs. convincing evidence, his affirmative defense that the victim's
PO3 FERDINAND FALLORINA Y death was caused by his gun accidentally going off, the bullet
FERNANDO, appellant. hitting the victim without his fault or intention of causing it;
hence, is exempt from criminal liability under Article 12,
paragraph 4 of the Revised Penal Code which reads
Article 248 of the Revised Penal Code, the penalty
for murder is reclusion perpetua to death. Since The following are exempt from criminal liability:
there is no modifying circumstance in the
commission of the crime, the appellant should be
sentenced to suffer the penalty of reclusion
4. Any person who, while performing a lawful act (b) Grave abuse of confidence;
with due care, causes an injury by mere accident (c) Nighttime;
without fault or intention of causing it. (d) Use of an motor vehicle ;
(e) Use of uperior strenght;
The basis for the exemption is the complete absence of intent (f) Cruelty.
and negligence on the part of the accused. For the accused to
Kidanapping with Murder as defined under Article 248 of
be guilty of a felony, it must be committed either with the Revised Penal Code, in relation to Article 267
criminal intent or with fault or negligence.33
The aggravating circumstance of abuse of superior strength
The elements of this exempting circumstance are (1) a is absorbed in treachery. The aggravating circumstance of
person is performing a lawful act; (2) with due care; (3) he nighttime (nocturnidad) cannot be absorbed in treachery
causes an injury to another by mere accident; and (4) because in this crimes," treachery arose from the defenseless
without any fault or intention of causing it. 34 An accident is position of Chua when he was killed, while nighttime was
an occurrence that "happens outside the sway of our will, purposely sought by the accused to facilitate immunity in the
and although it comes about through some act of our will, commission of the crime. The aggravating circumstance of
lies beyond the bounds of humanly foreseeable uninhabited place (despoblado) is also present, due to the
consequences." If the consequences are plainly foreseeable, it deliberate selection of an isolated place (Barrio Makatipo
Novaliches, Caloocan City) for killing and burying the victim
will be a case of negligence.
The aggravating circumstance of use of motor vehicle in the
The appellant committed murder under Article 248 of the commission of the crimes," can be considered present
Revised Penal Code qualified by treachery. because the Biscayne car of Ong was used to trail the victim's
car and to facilitate the commission of the crimes," and the
Under Article 248 of the Revised Penal Code, the penalty for escape of the accused.
murder is reclusion perpetua to death. Since there is no
modifying circumstance in the commission of the crime, the Evident premeditation attended the commission of the
appellant should be sentenced to suffer the penalty crimes, because the accused meditated, planned, and
of reclusion perpetua, conformably to Article 63 of the tenaciously persisted in the accomplishment of the crime.
Revised Penal Code.
Accused Ong was given the mitigating circumstances of plea
of guilty and one analogous to passion and obfuscation"
People VS. Ong because Chua previously threatened Ong for non-payment of
debt arising from gambling, causing Ong humiliation and
Intro: shame.
All contrary to law with the following generic aggravating "Conspiracy, connivance and unity of purpose and intention
circumstances: among the accused were present throughout in the execution
(a) Evident premeditations;
of this crime. The four participated in the planning and The only issue in this case, therefore, is whether or not the
execution of the crimes," and were at the scene in all its accused Ambrosio voluntarily participated in the
stages. They cannot escape the consequences of any of their commission of the crime.
acts even if they deviated in some detail from what they
originally thought of. Conspiracy implies concert of design
and not participation in every detail of the execution. Thus, Case #41
treachery should be considered against all persons
participating or cooperating in the perpetration of the
crime."

Brief Facts: PEOPLE OF THE PHILIPPINES, plaintiff-appellee,


That on or about April 23 to April 24, 1971, inclusive, in the vs. GEORGE CORTES y ORTEGA, accused-appellant.
municipality of Paraaque, province of Rizal, Philippines, [G.R. No. 137050. July 11, 2001]
and within the jurisdiction of this Honorable court, the
above named accused, being then private individuals,
conspiring and confederating together and mutually helping
one another, did then and there wilfully, unlawfully and with
*in this case the court took little discussions on the application
treachery and known premeditation and for the purpose of
killing one Henry Chua and thereafter extorting money from of the following aggravating circumstances as alleged in the
his family through the use of a ransom note, kidnapped and complaint.
carried away said Henry Chua, initially by means of friendly
gestures and later through the use of force, in an automobile,
and later after having taken him to an uninhabited place in
Caloocan City, with the use of force detained him (Henry Evident Premeditation.
Chua) and killed him in the following manner, to wit: The
accused after gagging and tying up Henry Chua and
In the aggravating circumstance of evident premeditation,
repeatedly threatening him with death, assured him that if
the prosecution must prove the following elements:
he would write and sign a ransom note for the payment by
his family of the sum of $50,000.00 (US), he would not be
killed and would be released upon receipt of the ransom (a) the time when the accused determined to commit
note, he was again gagged and tied up by the accused, and the crime,
thereafter stabbed in the abdominal region, several times
with an ice-pick, inflicting upon him (Henry Chua) mortal (b) an act manifestly indicating that the accused clung
wounds on his vital organs, which directly caused his death. to that determination, and
Brief Issue:
(c) a lapse of time between the determination and the (3) it facilitates the commission of the crime by insuring the
execution sufficient to allow the accused to reflect offender's immunity from identification or capture.
upon the consequences of the act
In the case at bar, no evidence suggests that accused
In the case at bar, the prosecution fails to prove this purposely sought the cover of darkness to perpetrate the
aggravating Circumstance crime, or to conceal his identity.

II. Cruelty Abuse of Superior Strength

For cruelty to be appreciated against the accused, it must be Abuse of superior strength is absorbed in treachery, so that
shown that the accused, for his pleasure and satisfaction, it cannot be appreciated separately as another aggravating
caused the victim to suffer slowly and painfully as he circumstance. Here, treachery qualified the offense to
inflicted on him unnecessary physical and moral pain. murder.

Rationale: The crime is aggravated because by deliberately Disregard of Sex


increasing the suffering of the victim the offender
denotes sadism and consequently a marked As to the aggravating circumstance of disregard of sex, the
degree of malice and perversity. same could not be considered as it was not shown that
accused deliberately intended to offend or insult the sex of
In the case at bar, The mere fact of inflicting various the victim, or showed manifest disrespect for her
successive wounds upon a person in order to cause his death, womanhood.
no appreciable time intervening between the infliction of one
(1) wound and that of another to show that he had wanted to In the case at bar, the accused mistook the victim for a
prolong the suffering of his victim, is not sufficient for taking man
this aggravating circumstance into consideration.
Intoxication
III
Nighttime
Ordinarily, intoxication may be considered either
Night-time becomes an aggravating circumstance only when: aggravating or mitigating, depending upon the
circumstances attending the commission of the crime.
(1) it is specially sought by the offender;
Intoxication has the effect of decreasing the penalty, if it is
(2) the offender takes advantage of it; or not habitual or subsequent to the plan to commit the
contemplated crime; on the other hand, when it is habitual guilty, mistaken identity and the alternative mitigating
or intentional, it is considered an aggravating circumstance. circumstance of intoxication.

A person pleading intoxication to mitigate penalty must 6 the trial court after considering the aggravating and
present proof of having taken a quantity of alcoholic mitigating circumstances attendant found the existence of
beverage prior to the commission of the crime, sufficient to the aggravating circumstances and appreciated only the
produce the effect of obfuscating reason. At the same time, mitigating circumstance of plea of guilty that was offset by
that person must show proof of not being a habitual drinker one of the aggravating circumstances. Hence, this review
and not taking the alcoholic drink with the intention to
reinforce his resolve to commit the crime

ISSUES:

FACTS: Whether the trial court erred in finding that the


aggravating circumstances of evident premeditation, cruelty,
1 an Information for murder (violation of Article 248 of the nighttime, abuse of superior strength, sex and intoxication
Revised Penal Code) against accused On George Cortes y attended the commission of the crime charged.
Ortega, for the murder of Edlyn S. Gamboa, a 16 year old girl

2 Accused admitted that he stabbed Edlyn.


HELD:
3 He entered a plea of guilty In virtue of his plea of guilty, the
trial court proceeded to satisfy itself of the voluntariness of The Solicitor General agrees with the accused that the
the plea by propounding questions to the accused to find out only aggravating circumstance present was treachery which
if he understood his plea and the legal consequence thereof. qualified the killing to murder and that there were two
mitigating circumstances of plea of guilty and intoxication,
4 The prosecution alleged that the aggravating circumstances not habitual. The penalty shall be reclusion perpetua, not
of evident premeditation, cruelty, nighttime, abuse of death, in accordance with Article 63 in relation to Article 248
superior strength, disrespect to sex, and intoxication were of the Revised Penal Code, as amended by Republic Act No.
present in the commission of the crime. 6759

5 The accused, on the other hand, raised the attendance of the


mitigating circumstances of voluntary surrender, plea of
CASE No. 42 On cross-examination, both Nerissa Tagala and Consuelo
People vs Regala Arevalo, separately testified that they saw the face of Regala,
G.R. No. 130508 April 5, 2000 despite of no electricity at the commission of the crime,
because he used a flashlight and took off the mask he was
FACTS: wearing, and thus, they remembered him wearing an earring
On the night of September 11, 1995, at Barangay Bangon in of his left ear, which he was still wearing at the time of the
Aroroy, Masbate, then 16-year old victim Nerissa Tagala and police line-up inside the police station.
her grandmother, Counselo Arevalo, were sleeping, when
appellant Armando Regala and his two other companions The trial court held that contradiction referred to a minor
entered the formers house. detail, cannot detract from the fact, that both Nerissa and
Consuelo positively identified the accused-appellant. As
Appellant and his companions entered the house through the correctly pointed out by the appellee, the victim was a 16-
kitchen and went to the room of the victims and poked at 8- year old barrio lass, not exposed to the ways of the world and
inch gun on them, one after the other, and hogtied both of was not shown to have any ill-motive to falsely implicate
them. Armando raped Nerissa in bed while her grandmother accused-appellant, who was a stranger. Hence, Dr. Ulandays
was hogtied on the floor. Later, she saw her grandmothers testimony does not support the contention of accused-
aparador being opened where two rings, two wrist watches, appellant that the victim voluntarily submitted to sexual
and money were taken from the aparador. After raping her in advances of Regala.
bed, Nerissa saw accused-appellant counting the money
taken from the aparador. Thereafter, she was brought to the The crime of robbery with rape was committed in 1995 when
kitchen, still hogtied and was raped again by the accused. RA 7659 was already in force. Under Article 294 of the
Revised Penal Code as amended, now provides, under
He was convicted in the lower court but accused-appellant paragraph 1 thereof: (1) The penalty of reclusion perpetua to
appealed his criminal case at the Regional Trial Court in death, when for any reason of or on occasion of the robbery,
Masbate. He questioned the sufficiency of the prosecutions the crime of homicide shall have been committed, or when
evidence in identifying him as one of the perpetrators of the the robbery shall have been accompanied by rape or
crime charged. And based on medico-legal, Dr. Conchita intentional mutilation or arson.
Ulanday, a health officer of Aroroy, testified herself that the
complaining witness either voluntarily submitted to a In this case, the additional rape committed by herein
sexual act or was forced into one. accused-appellant should not be considered as aggravating.
The penalty of reclusion perpetua imposed by the trial court
ISSUE: is proper. The judgment convicting Armando Regala y Abriol
(a) Whether additional rape committed in a crime of robbery guilty beyond reasonable doubt of the crime of Robbery with
be considered as an aggravating circumstance? Rape, where the victim is entitled to an additional award of
P50,000.00 as civil indemnity.

HELD:
PEOPLE vs BERDIDA Virgilio and Pedrito could ran away. Antonio and Federico
GR No. L-20183; June 30, 1966 were taken to a rail track where their hands were tied. They
were then further taken to a pier in North Harbor where they
DOCTRINE: were met by more armed men. The group of men beat
Rule of aggravating circumstances with regards to penalty Antonio and Federico until Antonio lost consciousness at
the presence of one generic aggravating circumstance, around 1:00am of the following day. The police, together
apart from the qualifying circumstance of treachery, suffices with Antonio's sister, found Antonio and Federico the
to fix the penalty for murder at the extreme punishment of following day. Antonio was still alive thus was rushed to the
death. hospital while Federico was already dead.
Nighttime from the facts and evidence of record in this
case, appellants took advantage of nighttime in committing ISSUE:
the felonies charged for they had evidently chosen to execute 1. Was the RTC correct in imposing the death penalty?
their victims under cover of darkness, at the dead of night, 2. Was the appreciation of the aggravating circumstances
when the neighborhood was asleep. proper?
Exception to the rule of absorbing Nighttime in Treachery
in as much as the treachery consisted in the fact that the
victims' hands were tied at the time they were beaten, the People vs Castillo
circumstance of nighttime is not absorbed in treachery, but PEOPLE OF THE PHILIPPINES, appellee, vs.
can be perceived distinctly therefrom, since the treachery ELIZABETH CASTILLO and EVANGELINE
rests upon an independent factual basis. A special case PADAYHAG, appellants.
therefore is present to which the rule that nighttime is [G.R. No. 132895. March 10, 2004]
absorbed in treachery does not apply. Doctrine/s and/or Rulings: We affirm the trial courts
Evident Premeditation from the time the accused judgment convicting Castillo. However, we acquit her co-
abducted the victims up to the time one of the latter lost accused Padayhag.
consciousness and the other died, sufficient time had lapsed To sustain a conviction for Kidnapping and Serious Illegal
Detention under Article 267 of the Revised Penal Code,[8]
for the accused to meditate and reflect on the consequences
the prosecution must establish the following: (1) the offender
of their act. is a private individual; (2) he kidnaps or detains another or
in any other manner deprives the victim of his liberty; (3) the
FACTS: act of kidnapping or detention is illegal; and (4) in the
At around 10:00pm, Antonio, Federico, Virgilio and Pedrito commission of the offense any of the following circumstances
had just left a store and were on their way to each of their is present: (a) the kidnapping or detention lasts for more
homes, when a group of men armed with bolos stopped them than three days; (b) it is committed by simulating public
and introduced themselves as policemen, saying that the four authority; (c) serious physical injuries are inflicted on the
are accused of killing someone and that they should go with victim or threats to kill are made; or (d) the person
them. Antonio and Federico held their hands up while
kidnapped or detained is a minor, female or a public officer. conspiracy, there must be a sufficient and unbroken chain of
[9] events that directly and definitely links the accused to the
Appellant Castillos Liability commission of the crime without any space for baseless
Castillo asserts that the victims parents did not pay her suppositions or frenzied theories to filter through.[26]
wages when she worked as a maid of the victims family.[10] Indeed, conspiracy must be proven as clearly as the
She claims that it was this injustice, her educational level and commission of the crime itself.[27]
her ignorance of the law, which impelled her to take Rocky. Conspiracy is established by the presence of two factors: (1)
She faults the trial court for refusing to consider this. Castillo singularity of intent; and (2) unity in execution of an
is mistaken. Whether or not her employer failed to pay her unlawful objective. The two must concur. Performance of an
salary is irrelevant. No amount of perceived injustice can act that contributes to the goal of another is not enough. The
serve as justification for any person to retaliate through the act must be motivated by the same unlawful intent. Neither
commission of another crime. The trial court was therefore joint nor simultaneous action is per se sufficient indicium of
correct in disregarding Castillos claim that Rockys parents conspiracy, unless proved to have been motivated by a
committed injustice on her. common design.[28]
Castillos claim of injustice cannot justify in any way her Padayhags act of fetching Rocky is not conclusive proof of
demand for ransom. Ransom is money, price or her complicity with Castillos plan, a plan Padayhag did not
consideration paid or demanded for redemption of a even know. Both appellants testified that Padayhag met
captured person or persons, a payment that releases from Castillo only because Castillo told Padayhag that Padayhags
captivity.[11] Thus, even if she had a right to demand boyfriend was sick. It was precisely on the pretext that they
payment of her unpaid wages, the money she actually were to visit Padayhags boyfriend that the two met. When
demanded and eventually received, is still ransom. they met, Padayhag realized that Castillo had deceived her:
Castillos reliance on her low educational level is similarly All these circumstances illustrate the absence of any hint of
unavailing. The penalty for kidnapping for ransom is the conspiracy. We also find that the prosecution failed to prove
singular and indivisible penalty of death. This bars the Padayhags guilt beyond reasonable doubt. In People v.
application of any alternative, mitigating or aggravating Gonzales[36] we held:
circumstance In the absence of conspiracy, if the inculpatory facts and
Appellant Padayhags Liability circumstances are capable of two or more explanations, one
The same cannot be said of Padayhag. Our review of the of which is consistent with the innocence of the accused and
evidence on record shows that the prosecution failed to the other consistent with his guilt, then the evidence does
prove Padayhags guilt beyond reasonable doubt. not fulfill the test of moral certainty and is not sufficient to
We reiterate the doctrine that an appeal in a criminal case support a conviction.
opens the entire case for review on any question including Penalty and Damages
those not raised by the parties.[25] This becomes even more Under Article 267 of the Revised Penal Code,[44] the penalty
imperative in cases where the penalty imposed is death. of death is imposed upon proof that the kidnapping was
There must be positive and conclusive evidence that committed to extort ransom from the victim or any other
Padayhag acted in concert with Castillo to commit the same person. We find that the prosecution has established
criminal act. To hold an accused guilty as a co-principal by Castillos guilt for this crime beyond reasonable doubt.
However, Castillos pecuniary liability must be modified to II. THE TRIAL COURT ERRED IN CONCLUDING
conform with jurisprudence. The award of exemplary THAT THERE WAS CONSPIRACY TO EXTORT
damages must be deleted in the absence of any aggravating RANSOM IN THIS CASE.
circumstance. Mr. Cebrero testified that their family suffered III. THE TRIAL COURT ERRED IN CONSIDERING
serious anxiety at the possibility of not seeing Rocky again. THE UNCOUNSELLED CONFESSION OF
[45] The pain and anguish they experienced justifies the EVANGELINE PADAYHAG.
award of moral damages. However, we reduce the trial courts IV. THE TRIAL COURT GRAVELY ERRED IN
award of moral damages to P100,000 in line with current IMPOSING THE DEATH PENALTY IN THE CASE AT BAR.
jurisprudence. [7]
Brief Facts: Before us on automatic review is the
Decision[1] of the Regional Trial Court of Paranaque, Branch People vs Tan, G.R. No. 132324. September 28, 1999
260, National Capital Judicial Region, in Criminal Case No.
95-86, finding appellants Elizabeth Castillo (Castillo) and Doctrines:
Evangeline Padayhag (Padayhag) guilty of Qualified Self defense
Kidnapping and Serious Illegal Detention[2] and sentencing When the accused invoke self-defense, the burden of proof is
them to death. shifted to them to prove that the killing was justified and that
The Information[3] charging Castillo, Padayhag and Imelda they incurred no criminal liability therefor. They must rely
Wenceslao with the crime of kidnapping, reads: on the strength of their own evidence and not on the
That on or about March 1, 1995, in Paraaque, Metro Manila, weakness of that of the prosecution, for even if the latter is
Philippines, and within the jurisdiction of the Honorable weak, it could not be disbelieved after their open admission
Court, said accused ELIZABETH CASTILLO and of responsibility for the killing.
EVANGELINE PADAYHAG, conspiring together, In the present case, it is incumbent upon Appellant Norly
confederating, and mutually helping one another, did then Tan to prove self-defense. Thus, he must prove that there
and there willfully, unlawfully and feloniously kidnap, carry was unlawful aggression on the part of the victim, that the
away, and seriously detain HORACIO CEBRERO IV @ means employed to prevent it were reasonable, and that
Rocky, a five years old child (sic), which kidnapping or there was lack of sufficient provocation o his part. However,
serious detention lasted for more than three (3) days thereby he failed to discharge this burden.
depriving him of his liberty, and which was committed for
the purpose of extorting ransom from the parents of the Crime and its Punishment
victim, to the damage and prejudice of the victim himself and The killing was attended by treachery; hence, the crime was
his parents. murder. The essence of treachery is the sudden and
Issues: unexpected attack, without the slightest provocation on the
I THE TRIAL COURT ERRED IN part of the person attacked. Treachery is present when the
MISAPPRECIATING (SIC) THE FACTS OF THE offender commits any of the crimes against persons,
CASE. employing means, methods or forms in the
execution thereof, which tend directly and especially to
insure its execution, without risk arising from the defense
which the offended party might make. In the case at bar, the On the 6th day of September, 1993, in Barangay Gatbo,
attack on Magdalino Olos was treacherous, because he was Municipality of Ocampo, Province of Camarines Sur,
caught off guard and was therefore unable to defend himself, Philippines, Norlito Tan and Jose Tan as an accomplice, with
as testified to by the prosecution witnesses and as indicated intent to kill, with treachery and evident premeditation,
by the wounds inflicted on him. conspiring, confederating together and mutually helping one
another, did then and there, wilfully, unlawfully and
Culpability of Jose Tan feloniously attack, assault, stone and stab with a deadly
The prosecution was not able to establish conspiracy in the weapon one Magdaleno Rudy Olos alias Modesto Olos,
killing of the victim; thus, Appellant Jose Tan is guilty only thereby inflicting upon the latter mortal wounds on the
as an accomplice. According to the widow Ofelio Olos, she different parts of his body which caused his death, to the
even heard Jose Tan telling and pleading with his brother to damage and prejudice of the heirs of the offended party in
stop his attack and stabbing of the victim. The most such amount as may be proven in court.
therefore that said accused could be liable for is merely that On December 14, 1995, Jose Tan was arrested in Ocampo,
of an accomplice, who, not being a principal cooperated in Camarines Sur. Subsequently, Norlito Tan was arrested on
the execution of the offense by previous and simultaneous April 1, 1996. They both pleaded not guilty.
acts, that in this case, by his stoning the victim Modesto Olos
and hitting him on the neck. However, the accused Jose Tans Issue:
act of stoning was neither a direct participation nor Whether or not self defense can be used as a justifying
indispensable to the killing of the victim. Also, as held by the circumstance in this case.
Supreme Court, when doubt exists whether an accused acted Whether Jose Tan can be considered as an accomplice in this
as principal or accomplice, the court should favor the lesser case.
or milder identity (People vs. Irenea, G.R. No. 44410, August
5, 1988).
The penalty of Appellant Jose Tan as an accomplice is one Case # 47 People v. Abarri
degree lower than that of the principal, which in murder G.R. No. 90185 March 1, 1995
cases is reclusion temporal, in its maximum period, to
death. Considering that he is entitled to the privileged Art. 8 Conspiracy and proposal to commit a felony
mitigating circumstance of minority, because he was only Art. 17 Principals
sixteen years old when the crime was committed, the trial
court should have lowered his penalty by two degrees, Doctrine/ Ruling:
i.e. prision correccional maximum to prision A conspiracy exists when two or more persons come to an
mayor medium. Likewise, he is entitled to the benefits of the agreement concerning the commission of a felony and decide
Indeterminate Sentence Law. to commit it. Proof of the agreement need not rest on direct
Since no aggravating or mitigating circumstance was proven, evidence, as the agreement itself may be inferred from the
the imposable penalty on Norlito Tan is reclusion perpetua. conduct of the parties disclosing a common understanding
among them with respect to the commission of the offense.
Facts:
The common intent of robbing the victim and committing showing that Cawaling had any moral influence over
the acts of lasciviousness can be inferred from their Andales.
behaviors.
Facts:
Abarri and Andales each poked a knife at Gan's neck and On October 14, 1988, at around 7:30 P.M., while Gregoria
forcibly brought her to the vacant lot. The other appellants Gan was walking along 4th Avenue, Kalookan City on her
followed them and watched while Abarri divested the victim way home, Ernesto Abarri and Ronnie Andales stopped her
of her valuables. After robbing the victim, Abarri with the use and each poked a knife at her neck. Abarri then grabbed
of a "balisong" tore the upper portion of the victim's blouse Gan's bag and warned: "Kung gusto mong mabuhay, huwag
and all the other appellants participated in removing her kang sisigaw." Gan was dragged by the two and brought
clothes, pawing her and biting her nipples. inside a fenced, vacant lot strewn with garbage and covered
The presence or absence of lewd designs is inferred from the with tall grass. Clemente Cawaling, Conrado Estrada and
nature of the acts themselves and the environmental Joselito Pajalago were former employees of Gan.
circumstances. We find that the acts of appellants in striping Once inside the vacant lot, Abarri, with the use of a
naked and hogtying the victim and touching her private parts "balisong, " tore the upper portion of Gan's blouse. The other
constitute lewd designs. accused then started tearing the rest of the blouse and
pulling down her pants. The torn blouse was used to tie her
However, in the case of Andales, the acts of lasciviousness mouth, hands and feet. When she was completely naked, the
committed by him culminated in the raping of the victim accused started touching her private parts.
when he was left alone with her. Nothing in the records show Abarri opened Gan's handbag and took a bunch of keys,
that the other accused had knowledge or were aware of the which included the key for her store at Carmen Planas Street
rape committed by Andales. Consequently, he alone is guilty in Binondo, Manila. He also got her watch valued P2,000.00,
of robbery with rape. necklace valued at P5,000.00 and wallet containing
P250.00.
Likewise, we do not regard the remarks made by
Cawaling to Andales as sufficient to make him a After robbing Gan, appellants left except Andales.
principal by inducement or a co-conspirator. Before Before leaving, Cawaling told Andales: "Nognog,
a remark can produce such an effect, the same must (referring to Andales) bahala ka na, sampung taon
be of a nature and uttered in such a manner as to na rin na hindi nakakatikim 'yan, makatas pa 'yan."
become the determining cause of the crime, as to
make the utterance a command from a superior to a Andales then dragged Gan to a dark spot and after loosening
subordinate. In the case at bench, it appears that the the tie on her legs, raped her twice. After satisfying his lust,
decision of Andales to rape the victim had been Andales left.
made before Cawaling uttered the remarks.
Cawaling was then leaving the place with Abarri, Gan waited for about 20 minutes before she started to roll
Estrada and Pajalago while Andales purposely over to the middle of the lot. In the process, the tie on her
stayed behind with the victim. There is not even a mouth loosened and she was able to shout for help.
Responding to her cries, neighbors came and untied her corresponding period, depending upon the modifying
hands. circumstances present, but never exceeding 12 years of
prision mayor, and (2) for the crime of murder, reclusion
Meanwhile, at around 8:30 P.M. of the same day, Barangay temporal in its maximum period to death, depending upon
Captain Anita Alejo was informed by a resident that the modifying circumstances present. in other words, in the
somebody was opening the store of Gan. Repairing at the absence of aggravating circumstances, the extreme penalty
place, Alejo saw Abarri and Estrada. She noticed that the could not be imposed upon him. However, under Article 48
door of the store had been partly opened. When she asked said penalty would have to be meted out to him, even in the
the two what they were doing there, Abarri answered that absence of a single aggravating circumstance. Thus, said
Gan instructed them to get the latter's pants. Alejo brought provision, if construed in conformity with the theory of the
them to the barangay hall for investigation. Upon further prosecution, would be unfavorable to the movant.
questioning, Abarri admitted to forcibly bringing Gan to a
vacant lot and binding her arms and legs. Alejo turned over Facts:
the two to the custody of the police detachment in Binondo. A warrant was issued charging Senator Enrile, the spouses
Rebecco and Erlinda Panlilio, and Gregorio Honasan with
The police brought Abarri and Andales to the crime scene. the crime of rebellion with murder and multiple frustrated
However, Gan was no longer there when they arrived. The murder allegedly committed during the period of the failed
police proceeded to Gan's house where the latter positively coup attempt from November 29 to December 10, 1990. The
identified the two as among those persons who robbed her. petitioner's counsel contended that the petitioner is charged
with a crime that does not exist in the statute books,
Issue: therefore, the same must be dismissed as it is just a mere
Are the accused guilty of robbery with rape? flight of rhetoric.

Issue:
Case No. 48 Whether or not under Article 48 of the Revised Penal Code
Enrile vs. Salazar rebellion may properly be complexed with a common
186 SCRA 217 offense, murder.
G.R. No. 92163 64
June 5, 1990
#49 Santiago v Garchitorena
Intro:
Article 48 of our Penal Code cannot be applied in the case at ART 10 RPC - Supplementing Special Laws
bar. If murder were not complexed with rebellion, and the
two crimes were punished separately (assuming that this CONTINUING CRIME - For delito continuado to exist there
could be done), the following penalties would be imposable should be a plurality of acts performed during a period of
upon the movant, namely: (1) for the crime of rebellion, a time; unity of penal provision violated; and unity of criminal
fine not exceeding P20,000 and prision mayor, in the intent or purpose, which means that two or more violations
of the same penal provisions are united in one and same Program. In this respect, and responding directly to the
instant or resolution leading to the perpetration of the same concerns of the accused through counsel, the prosecution is
criminal purpose or aim categorical that there will not be 32 accusations but only
one . . . (
a delito continuado consists of several crimes but in reality
there is only one crime in the mind of the perpetrator. ISSUE:
Is the consolidation of cases proper for violation of EO
FACTS: 324? / Was there only one offense committed?

Petitioner was charged with performing a single criminal act RULING:


that of her approving the application for legalization of 32 YES. Under Article 10 of the Revised Penal Code, the Code
aliens not qualified under the law to enjoy such privilege shall be supplementary to special laws, unless the latter
under EO 324, causing undue injury to one offended party, provide the contrary. Hence, legal principles developed from
the Government, done on a single day, i.e., on or about the Penal Code may be applied in a supplementary capacity
October 17, 1988. to crimes punished under special laws.

According to petitioner, unless she was furnished with the Applying the concept of delito continuado
names and identities of the aliens, she could not properly
plead and prepare for trial. The 32 Amended Informations aver that the offenses were
committed on the same period of time, i.e., on or about
The information was then amended reproducing verbatim October 17, 1988. The strong probability even exists that the
the allegation of the original information, except that instead approval of the application or the legalization of the stay of
of the word "aliens" in the original information each the 32 aliens was done by a single stroke of the pen.
amended information states the name of the individual Likewise, the public prosecutors manifested at the hearing
whose stay was legalized. the motion for a bill of particulars that the Government
suffered a single harm or injury.
At the hearing of the motion for a bill of particulars, the
public prosecutors manifested that they would file only one Criminal Case No. 16698 is MODIFIED in the sense that the
amended information embodying the legalization of stay of Office of the Special Prosecutor of the Office of the
the 32 aliens. As stated in the Order dated November 12, Ombudsman is directed to consolidate the 32 Amended
1992 of the Sandiganbayan (First Division): Informations into one information charging only one offense
under the original case number,
On the matter of the Bill of Particulars, the prosecution has
conceded categorically that the accusation against Miriam #50 People vs. Dela Torre
Defensor Santiago consists of one violation of the law Rule 122 Sec. 1 of the 2000 Rules of Criminal Procedure
represented by the approval of the applications of 32 foreign provides, any party on appeal from a judgment or final order,
nationals for availment (sic) of the Alien Legalization unless the accused will be placed in double jeopardy.
People v. Leones (GR Nos. 128514 & 143856-61), it Issue: WON an increase in the penalty imposed by the lower
unmistakably declared that "while it is true that this Court is court will violate the right of the accused against double
the Court of last resort, there are allegations of error jeopardy?
committed by a lower court which we ought not to look into
to uphold the right of the accused. Such is the case in an Case # 52
appeal by the prosecution seeking to increase the penalty Cristobal v. Labrador
imposed upon the accused for this runs afoul of the right of G.R. No. L-47941 December 7, 1940
the accused against double jeopardy."
Heirs of Tito Rillorta vs. Firme (157 SCRA 518) clarified the ABSOLUTE PARDON
Kepner Doctrine stating that "an appeal of the prosecution DOCTRINE: Absolute pardon not only blots out the
from a judgment of acquittal (or for the purpose of crime committed but removes all disabilities resulting from
increasing the penalty imposed upon the convict) would the conviction.
place him in double jeopardy" FACTS: The CFI of Rizal found respondent Santos guilty of
The ban on double jeopardy is deeply rooted in estafa, however, he continued to be a registered elector in the
jurisprudence. The doctrine has several avowed purposes. city of Malabon, Rizal and was seated as the municipal
Primarily, it prevents the State from using its criminal president from 1934 to 1937. On 1938, Commonwealth Act
processes as an instrument of harassment to wear out the No 357 or the Election Code was approved which disqualifies
accused by a multitude of cases with accumulated trials. It the respondent from voting for having been declared by
also serves the additional purpose of precluding the State, final judgment guilty of any crime against property. The
following an acquittal, from successively retrying the respondent applied to the President for an absolute pardon
defendant in the hope of securing a conviction. And finally, it and granted the petition restoring the respondent to his full
prevents the State, following conviction, from retrying the civil and political rights, except that with respect to the right
defendant again in the hope of securing a greater penalty. to hold public office or employment, he will be eligible
The Court also states: "Whatever error may have been for appointment only to positions which are clerical or
committed by the lower court was merely an error of manual in nature and involving no money or property
judgment and not of jurisdiction. It did not affect the responsibility.
intrinsic validity of the decision. This is the kind of error that
can no longer be rectified on appeal by the prosecution no On 1940, Cristobal filed a petition for the exclusion of the
matter how obvious the error may be." name of Santos from the list of voters in precinct no. 11 of
Facts: Wilfredo dela Torre was charged and convicted of 2 Malabon, Rizal on the ground that the latter is disqualified.
counts of acts of lasciviousness and 4 counts of rape against After hearing, the court denied the petition for exclusion and
his daughter Mary Rose dela torre. declared that the pardon extended in favor of respondent has
The RTC rendered its judgment imposing the penalty of had the effect of excluding him from the disqualification
reclusion perpetua, maintaining there were circumstances created by the Election Code.
mitigating the gravity of the offense. An appeal was made
seeking to increase the penalty from reclusion perpetua to
the supreme penalty of death.
ISSUE: Whether or not the pardon not only blots out the of the arrest, the escapee is in the continuous act of
crime committed but removes all disabilities resulting from committing a crime evading the service of his sentence.
the conviction? ISSUE: Whether or not the Court of First Instance of Manila
has jurisdiction to hear and decide the case against Parulan?
FACTS: Petitioner Parulan was confined in the state
54. RICARDO PARULAN, petitioner, vs. DIRECTOR penitentiary at Muntinlupa, Rizal, serving a sentence of life
OF PRISONS, respondent. G.R. No. L-28519 imprisonment which, however, was commuted to 20 years by
February 17, 1968 the President. He was then transferred to Fort Bonifacio in
Ruling: There are crimes which are called transitory or Makati. In October 1964, while still serving his sentence, he
continuing offenses because some acts material and essential escaped. He was eventually recaptured in Manila.
to the crime occur in one province and some in another, in Consequently, he was prosecuted for the crime of evasion of
which case, the rule is settled that the court of either service of sentence and was found guilty. As a recourse, he
province where any of the essential ingredients of the crime filed a petition for the issuance of writ of habeas corpus
took place has jurisdiction to try the case. There are, also, directed against respondent Director of Prisons.
crimes which although all the elements thereof for its
consummation may have occurred in a single place, yet by
reason of the very nature of the offense committed, the
violation of the law is deemed to be continuing. The crime of
PEOPLE OF THE PHILIPPINES vs. RENATO
evasion of service of sentence is an example of the latter kind
of crime - when the prisoner in his attempt to evade the TALUSAN y PANGANIBAN
service of the sentence imposed upon him by the courts and
thus defeat the purpose of the law, moves from one place to G.R. No. 179187 July 14, 2009
another; for, in this case, the act of the escaped prisoner is a
continuous or series of acts, set on foot by a single impulse CARPIO MORALES, J.:
and operated by an unintermittent force, however long it
may be. It may not be validly said that after the convict shall
have escaped from the place of his confinement the crime is
Doctrine(s): Special complex crime. Where the law
fully consummated, for, as long as he continues to evade the
provides a single penalty for two or more component
service of his sentence, he is deemed to continue committing
offenses, the resulting crime is called a special complex
the crime, and may be arrested without warrant, at any place
crime. In a special complex crime, the prosecution must
where he may be found. Rule 113 of the Revised Rules of
necessarily prove each of the component offenses with the
Court may be invoked in support of this conclusion, for,
same precision that would be necessary if they were made
under section 6[c] thereof, one of the instances when a
the subject of separate complaints.
person may be arrested without warrant is where he has
Mitigating Circumstance of voluntary plea of guilty
escaped from confinement. Undoubtedly, this right of arrest
in single and indivisible penalty. The presence of
without a warrant is founded on the principle that at the time
mitigating circumstance of voluntary plea of guilty has no
effect or disregarded incase the penalty imposable is single An information for kidnapping with rape was filed against
and indivisible. appellant by on the basis of the report submitted by the
Automatic Review in case the penalty imposed is medico legal.
death, reclusion perpetua or life imprisonment.
When the penalty of death, reclusion perpetua or life Upon arraignment, appellant, with the assistance of his
imprisonment is imposed, the case is automatically
counsel de oficio, entered a plea of guilty. The lower court
forwarded to the Supreme Court on automatic review.
thereupon conducted a searching inquiry into the
However, such automatic review to the Supreme Court does
not bar the referring of the case to the Court of Appeals for voluntariness of appellants plea, and despite repeated
intermediate disposition. questions and just as repeated answers showing that
Searching inquiry incase of plea of guilty in a crime appellant understood his plea and its consequences, the trial
having a penalty of death, reclusion perpetua or life court still ordered the prosecution to, as it did, present
imprisonment. There is thus no hard and fast rule as to evidence.
how a judge may conduct a "searching inquiry." As long as
the voluntary intent of the accused and his full Issue: Whether or not a searching inquiry is
comprehension of the consequences of his plea are required in case of plea of guilty where the penalty
ascertained. imposed is death, reclusion perpetua or life
imprisonment?

Facts: In January 14, 2004, while AAA was on her way to


school, appellant kidnapped AAA by deceiving the latter that G.R. No. 198554 July 30, 2012
they would would go to Jollibee, but in fact appellant MAJOR GENERAL CARLOS F. GARCIA, AFP (RET.),
brought AAA to a house in Imus, Cavite. AAA was thereafter Petitioner,
under appellants control and custody for eight days during vs.
which he abused her by inserting his finger inside her vagina THE EXECUTIVE SECRETARY,
on a daily basis before breakfast, despite her resistance.
Doctrines:
AAA having failed to return home, her stepfather BBB went 1 A court-martial case is a criminal case and the
to search for the former. A neighbor then informed him that General Court Martial is a court akin to any other
he saw appellant with AAA while the latter was on her way to courts. The General Court Martial is a court within
the strictest sense of the word and acts as a criminal
school. At dawn of the following day, January 23, 2004,
court. On that premise, certain provisions of the
appellant, who was with AAA, was apprehended.
Revised Penal Code, insofar as those that are not
provided in the Articles of War and the Manual for
Courts-Martial, can be supplementary. Under Article
10 of the Revised Penal Code: Art. 10. Offenses not arbitrary discrimination, whether occasioned by the
subject to the provisions of this Code.Offenses express terms of a statute or by its improper
which are or in the future may be punishable under execution through the states duly-constituted
special laws are not subject to the provisions of this authorities
Code. This Code shall be supplementary to such
laws, unless the latter should specially provide the Facts:
contrary. Major General Carlos Garcia of the AFP was charged in a
Court Martial for failure to disclose all his existing assets in
2 A special law is defined as a penal law which his Sworn Statement of Assets and Liabilities and Net Worth
punishes acts not defined and penalized by the for the years 2002 and 2003. After six (6) years and two (2)
Revised Penal Code. In the present case, petitioner months of preventive confinement, Garcia was found guilty
was charged with and convicted of Conduct of the charged offenses and was dishonorably discharged
Unbecoming an Officer and Gentleman (96th Article from the service, forfeited all pay and allowances due and
of War) and Violation of the 97th Article of War, or confined at hard labor at such place the reviewing authority
Conduct Prejudicial to Good Order and Military may direct for a period of two (2) years.
Discipline, both of which are not defined and Issue: WON the period of preventive confinement of Garcia
penalized under the Revised Penal Code. The shall be credited to the sentence imposed by the court
corresponding penalty imposed by the General Court martial.
Martial, which is two (2) years of confinement at Ruling: Yes. Applying the provisions of Article 29 of the
hard labor is penal in nature. Therefore, absent any Revised Penal Code, the time within which the petitioner was
provision as to the application of a criminal concept under preventive confinement should be credited to the
in the implementation and execution of the General sentence confirmed by the Office of the President. The period
Court Martial's decision, the provisions of the of confinement of six years shall be credited in his favor and
Revised Penal Code, specifically Article 29 should be deducted from the two (2) years to which the accused was
applied. sentenced.

3 Article 29 of the Revised Penal Code in the Articles


of War is in accordance with the Equal Protection FERDINAND A. CRUZ, Petitioner, v. THE PEOPLE
Clause of the 1987 Constitution. According to a long OF THE PHILIPPINES, Respondnet.
line of decisions, equal protection simply requires G.R. No. 176504, Spetember 03, 2008
that all persons or things similarly situated should
be treated alike, both as to rights conferred and ARTICLE
responsibilities imposed. It requires public bodies Article 310 of the Revised Penal Code, theft is
and institutions to treat similarly situated qualified when it is, among others, committed with
individuals in a similar manner. The purpose of the grave abuse of confidence, to wit:
equal protection clause is to secure every person
within a states jurisdiction against intentional and
ART. 310. Qualified theft. - The crime of with grave abuse of confidence, steal and carry away the
theft shall be punished by the penalties next amount of PhP15,000.00. He entered a plea of not guilty
higher by two degrees than those during arraignment, and the witnesses for the prosecution
respectively specified in the next preceding testified that he received the said amount from Hemisphere-
article, if committed x x x with grave abuse Leo Burnett and issued a receipt but failed to turn over the
of confidence x x x. amount to Porta-Phone Rentals. The prosecution changed
the case to Qualified Theft.
The settled rule is that when an accused pleads to Ferdinand Cruz in his defense alleged that he issued an
the charge, he is deemed to have waived the right acknowledgment receipt to Hemisphere and remitted the
to preliminary investigation and the right to amount to the accounting supervisor. He was asked by the
question any irregularity that surrounds it. supervisor to sign the official receipt because he was the one
who closed the deal.
Under Article 310 of the Revised Penal Code, the The RTC found Ferdinand guilty beyond reasonable doubt
penalty for Qualified Theft is two degrees for the crime of QUALIFIED THEFT. He filed a Motion for
higher than that specified in Article 309. New Trial on the grounds of (1) absence of a preliminary
investigation for the crime of qualified theft and (2) newly
FACTS: discovered evidence the testimony of a former employee of
Ferdinand Cruz was a Marketing Manager of Porta-Phone Hemisphere testifying on Ferdinands signing of an
Rentals, Inc., a corporation engaged in the lease of cellular acknowledgment receipt. The RTC revived and reinstated the
phones. He went to his office, obtained a pad of official conviction of Ferdinand. Upon appeal, the Court of Appeals
receipts from the collection officers table. He delivered the affirmed the RTC Decision.
communication equipment and received the PhP15,000.00
payment and issued a receipt even though he not authorized ISSUE:
to receive cash payments and issued receipts. He then failed 1 Whether Ferdinand was denied of due process when
to deliver the cash to his office. When he was confronted, he was indicted for qualified theft even as he was
Ferdinand admitted that he deposited the amount to his initially investigated for estafa/falsification of
personal bank account. He was instructed to remit the private documents.
amount which he failed to do claiming that the company has 2 Whether Ferdinands guilt was not established
paid his reimbursements.. The company sent a demand beyond reasonable doubt.
letter which he answered stating that he already remitted the 3 Whether the Indeterminate Sentence Law should be
amount to the accounting supervisor which the latter denied. applied.
He refused to turn over the amount despite demands made
by the officers of the company which prompted the company
to file the a case of Estafa/Falsification of Documents. HELD:
According to the Information filed before the RTC of Makati, 1 Ferdinand was not denied of due process. The
Ferdinand Cruz as a Marketing Manager of Porta-Phone settled rule is that when an accused pleads to the
Rentals, Inc. had access to the funds of the corporation and charge, he is deemed to have waived the right to
preliminary investigation and the right to question degrees than those respectively specified in the next
any irregularity that surrounds it. In the instant case, preceding article, if committed x x x with grave
Ferdinand did not present evidence that abuse of confidence x x x.
arraignment was forced upon him. On the contrary,
he voluntarily pleaded to the charge and actively The prosecution established, beyond the shadow of
participated in the trial of the case doubt that Ferdinand took and kept the fifteen
thousand peso-collection from the companys client.
It is not correct for Ferdinand to claim that
preliminary investigation on the charge of qualified 3 The RTC imposed on petitioner the indeterminate
theft was not accorded him. The truth is, Ferdinand penalty of 10 Years and 1 Day of prision mayor as
was able to answer the initial charge of minimum to 14 Years, 8 Months and 1 Day of
estafa/falsification of private documents through his reclusion temporal, as maximum.
counter-affidavits. Based on the same complaint
affidavit and the same sets of evidence presented by Under Article 310 of the Revised Penal Code, the
the complainant, the prosecutor deemed it proper to penalty for Qualified Theft is two degrees higher
charge Ferdinand with qualified theft. Since the than that specified in Article 309. Paragraph 1 of
same allegations and evidence were proffered by the Article 309 provides that if the value of the thing
complainant in the qualified theft, there is no need stolen is more than P12,000.00 but does not exceed
for Ferdinand to be given the opportunity to submit P22,000.00, the penalty shall be prision mayor in its
counter-affidavits anew, as he had already answered minimum and medium periods. In this case, the
said allegations when he submitted counter- amount stolen was P15,000.00. Two degrees higher
affidavits for the original indictment of than prision mayor minimum and medium is
estafa/falsification of private documents. reclusion temporal in its medium and maximum
periods. Applying the Indeterminate Sentence Law,
2 The elements of the crime of theft are the following: the minimum shall be prision mayor in its
(1) there was a taking of personal property; (2) the maximum period to reclusion temporal in its
property belongs to another; (3) the taking was minimum period or within the range of 10 years and
without the consent of the owner; (4) the taking was 1 day to 14 years and 8 months. There being neither
done with intent to gain; and (5) the taking was aggravating nor mitigating circumstance in the
accomplished without violence or intimidation commission of the offense, the maximum period of
against the person or force upon things.12 Under the indeterminate sentence shall be within the range
Article 310 of the Revised Penal Code, theft is of 16 years, 5 months and 11 days to 18 years, 2
qualified when it is, among others, committed with months and 20 days. The minimum penalty imposed
grave abuse of confidence, to wit: by the RTC is correct. However, the maximum
period imposed by RTC should be increased to 16
ART. 310. Qualified theft. - The crime of theft shall years, 5 months and 11 days.
be punished by the penalties next higher by two
As already stated, he is presumed to be the author of the
Case No. 58 falsification because he was in possession of the forged
PEOPLE OF THE PHILIPPINES vs. LICERIO P. vouchers and he used them in order to receive public monies
SENDAYDIEGO, JUAN SAMSON & ANASTACIO from the provincial treasurer.
QUIRIMIT He is a co-principal in the six crimes of malversation because
G.R. No. L-33254 & G.R. No. L-33253 January 20, 1978 he conspired with the provincial treasurer in committing
those offenses. The trial court correctly ruled that a private
DOCTRINE & DECISION: person conspiring with an accountable public officer in
The crimes committed in these three cases are not committing malversation is also guilty of malversation.
complex. Separate crimes of falsification and malversation
were committed. These are not cases where the execution of FACTS:
a single act constitutes two grave or less grave felonies or In these three cases of malversation through
where the falsification was used as a means to commit falsification, the prosecution's theory is that in 1969 Licerio
malversation. P. Sendaydiego, the provincial treasurer of Pangasinan, in
In the six vouchers the falsification was used conspiracy with Juan Samson y Galvan, an employee of a
to conceal the malversation. It is settled that if the lumber and hardware store in Dagupan City, and with
falsification was resorted to for the purpose of hiding the Anastacio Quirimit, the provincial auditor, as an accomplice,
malversation, the falsification and malversation are separate used six (6) forged provincial vouchers in order to embezzle
offenses from the road and bridge fund the total sum of P57,048.23.
In the instant case, the provincial , as the custodian Paragraph 4 is a certification of Voucher No. 10724
than of the money forming part of the road and bridge could dated February 28, 1969 certifying that the voucher has been
have malversed or misappropriated it without falsifiying any pre-audited and signed by the auditor. Paragraph 5 is a
voucher. The falsification was used as a device to prevent certification signed by the provincial treasurer that
detection of the malversation. the account mentioned in the provincial engineer's
Falsification and malversation did not constitute a certification "was paid in the amount and on the date shown
complex crime because the falsifications were not necessary below and is chargeable as shown in the summary hereof. " It
means for the committing malversations. Each falsification may be noted that the provincial treasurer signs two part of
and each malversation constituted independent offenses the voucher.
which must be punished separately. The falsifications cannot According to the prosecution, Samson also signed on
be regarded as constituting one continuing offense impelled the left margin of the six vouchers below the stamped words:
by a single criminal impulse. Each falsification of a voucher "Presented to Prov. Treasurer. By Juan Samson."
constitutes one crime. The falsification of six vouchers After trial the lower court acquitted the auditor,
constitutes six separate or distinct offenses. And each Quirimit and found Sendaydiego and Samnson guilty of
misappropriation as evidenced by a provincial voucher malversation through falsification of public or official
constitutes a separate crimes of malversation were documents
committed. Appellant Samson is a co-principal in each of the
said twelve offenses.
Sendaydiego and Samson appealed to this Court. element, the accused cannot be held liable for kidnapping. In
Sendaydiego died on October 5, 1976. His appeal as to his the present case, the prosecution merely proved that
criminal liability was dismissed. appellant forcibly dragged the victim toward a place only he
knew. There being no actual detention or confinement, the
appellant may be convicted only of grave coercion.
ISSUE:
Whether or not appellants are liable for the crimes of
falsicification of public documents and six crimes of
malversation?

No Motive to "Kidnap"

Motive is not an element of the crime. Furthermore, motive


becomes material only when the evidence is circumstantial
or inconclusive, and there is some doubt on whether a crime
has been committed or whether the accused has committed
PEOPLE OF THE PHILIPPINES v. ARNULFO
it. Indeed, motive is totally irrelevant when ample direct
ASTORGA
evidence sustains the culpability of the accused beyond
reasonable doubt. In this case, the identity of appellant is not
G.R. No. 110097, 22 December 1997 in question. He himself admitted having taken Yvonne to
Maco Central Elementary School.

Introduction (Provisions involved, principles,


jurisprudence): Kidnapping or Coercion?

Actual detention or "locking up" is the primary element of The prosecution failed to prove one essential element of
kidnapping. If the evidence does not adequately prove this kidnapping the fact of detention or the deprivation of
liberty. Under Article 267 of the Revised Penal Code, the The Spanish version of Article 267 of the Revised Penal Code
elements of kidnapping are as follows: uses the terms "lockup" (encerrar) rather than "kidnap"
(secuestrar or raptar). Lockup is included in the broader
term of "detention," which refers not only to the placing of a
person in an enclosure which he cannot leave, but also to any
1 That the offender is a private individual. other deprivation of liberty which does not necessarily
involve locking up. Likewise, the Revised Penal Code was
originally approved and enacted in Spanish. Consequently,
2 That he kidnaps or detains another, or in any
the Spanish text is controlling in cases of conflict with the
other manner deprives the latter of his liberty.
English version, as provided in Section 15 of the Revised
Administrative Code.
3 That the act of detention or kidnapping must
be illegal.

4 That in the commission of the offense, any of the


A review of the events as narrated by the prosecution
following circumstances is present:
witnesses ineluctably shows the absence of "locking up." It is
clear that the appellant and the victim were constantly on the
1 That the kidnapping or detention lasts move. There is no actual confinement or restraint of the
for more than five (5) days; or victim, which is the primary element of
kidnapping. Appellant's apparent intention was to take
2 That it committed simulating public Yvonne against her will towards the direction of Tagum. The
authority; or evidence does not show that appellant wanted to detain
Yvonne; much less, that he actually detained her. Appellant's
3 That any serious physical injuries are forcible dragging of Yvonne to a place only he knew cannot
inflicted upon the person kidnapped or be said to be an actual confinement or restriction on the
detained or threats to kill him are made; person of Yvonne. There was no "lockup." Accordingly,
or appellant cannot be convicted of kidnapping under Article
267 of the Revised Penal Code.
4 That the person kidnapped or detained
is a minor, female, or a public officer.

Rather, the felony committed in this case is grave coercion


under Article 286 of the same code. Grave coercion
or coaccion grave has three elements: (a) that any person is
prevented by another from doing something not prohibited
by law, or compelled to do something against his or her will,
be it right or wrong; (b) that the prevention or compulsion is
effected by violence, either by material force or such a While the accused and Yvonne were walking in the situation
display of it as would produce intimidation and, as described, somewhere near the Luponlupon bridge, they
consequently, control over the will of the offended party; and met some group of youngsters-men and the said group was
(c) that the person who restrains the will and liberty of suspicious about the man who was bringing a child, thus the
another has no right to do so or, in other words, that the said group decided to follow them. Accused hurriedly walked
restraint is not made under authority of a law or in the fast with Yvonne, and to prevent from being overtaken, he
exercise of any lawful right. When appellant forcibly dragged carried the victim and ran but they were chased and were
and slapped Yvonne, he took away her right to go home to overtaken.
Binuangan. Appellant presented no justification for
preventing Yvonne from going home.

Edwin Fabila declared that Jonathan, one of his companions


with others in chasing, asked the accused where they were
bound and he answered towards Binuangan but the group
noticed something suspicious because their destination was
Brief facts: already towards Tagum which is an opposite direction to
Binuangan.

While there was a brown out, accused or "Boy" Astorga told


Yvonne Traya, who was only eight (8) years old at that time Jonathan, one of those who chased, knew Yvonnes family
and who stays with her grandparents and so with her parents and he got from the accused Yvonne who showed some
at Sitio Binuangan, Maco, to go with him to buy candy and resistance, and the group brought the accused and Yvonne to
immediately grabbed and held Yvonnes hand as the latter Yvonnes home at Binuangan.
did not answer, placed his hand on her shoulder, covered her
mouth, and went and walked inside the compound of Maco
Elementary School, and later, there being no person around
the gate, accused brought her out to the highway and walked
towards the direction of Tagum which is the opposite
direction towards her grandparent's house at Binuangan,
Brief issues:
hence Yvonne cried and protested that she must go home but
the accused did not heed her plea.
Whether the accuseds motive to kidnap Yvonne Traya is
relevant to convict him for the crime of kidnapping. No,
motive is not relevant.
ORTEGA vs PEOPLEGR No. 151085August 20, 2008

FACTS:
The petitioner, Joemar Ortega, who was then 14 years old, was
Whether Yvonne Traya was not detained, locked-up or charged with the crime of rape
deprived of her liberty so as not to convict the accused for the for allegedly raping AAA, who was about 8 years old. That the rape
crime of kidnapping but only for grave coercion. Yes! happened in 3 occasions. The RTC ruled that the petitioner is
guilty beyond reasonable doubt in the crime of rape and is
sentenced to reclusion temporal. The CA affirmed the ruling of the
trial court. During the pendency of the case in the SC, RA 9344
Juvenile Justice and Welfare Act was enacted that establishes a
comprehensive system to manage children in conflict with the law. At
the case at bar, because the petitioner was a minor under 15 years of
age at the commission of the crime, he can be relieved from criminal
liability.

ISSUE:
ORTEGA V. PEOPLE (CRIMINAL) Whether or not the petitioner can avail exempting circumstance
provided by the newly enacted law on minors in conflict with
ARTICLE 12 - EXEMPTING CIRCUMSTANCES law.
HELD
Section 6 of RA 9344 clearly and explicitly provides: : Yes
RATIO
Section 6. Minimum Age of Criminal Responsibility - A child : The petitioner can avail the exempting circumstance that will relieve
15 years of age or under at the time of the commission of the him from criminal liability because the law enacted was favorable to
offense shall be exempt from criminal liability. However, the the accused, and is therefore retroactive in application. Juvenile
child shall be subjected to an intervention program pursuant Justice and Welfare Act provides that a child under 15 years of age in
to Section 20 of this Act. the commission of the offense shall be exempt from criminal liability,
but is subject to an intervention program. Exemption from criminal
A child above 15 years but below 18 years of age shall liability, however, does not include exemption from civil liability.
likewise be exempt from criminal liability and be subjected Section 64 of the newly enacted law also provides that cases of
to an intervention program, unless he/she acted with children under 15 years of age at the commission of the crime, shall
discernment, in which case, such child shall be subjected to immediately be dismissed and the child shall be referred to
the appropriate proceedings in accordance with this Act the appropriate local social welfare and development officer.
The Court therefore held that the case against Joemar Ortega the accused as a friend, almost like a relative, as in fact she
is hereby DISMISSED. Petitioner is hereby referred to the local called him Tito Loloy.
social welfare and development officer of the locality for the
appropriate intervention program As correctly observed by the Solicitor General: (A)s regards
the acts imputed to Estela, the delay of seven (7) days from
the date of her knowledge of the rape incident on 4 April
Case No. 65 1992 in reporting to the authorities the rape of her daughter
is excusable. At that time, she was not yet certain of the steps
PEOPLE OF THE PHILIPPINES vs. IRVIN she would take considering the delicate nature of the
TADULAN y EPAN problem they were facing (citing People v. Danguilan, 218
G.R. No. 117407 | April 15, 1997 SCRA 98; People v. Joaquin, Jr., 225 SCRA 179). Besides, we
PADILLA, J. have ruled that a delay in prosecuting the rape is not
indicative of fabricated charges.
DOCTRINE / RULING:
Alibi; It has been held time and again that for alibi to Pardon; It is clear to the mind of this Court that the
prosper as a defense the accused must show that he was so complainant has not expressly pardoned the said accused.
far away that he could not have been physically present at Besides, there are authorities holding that pardon must be
the place of the crime, or its immediate vicinity at the time of granted not only by the parents of an offended minor but
its commission (People vs. Tasurra, 192 SCRA 266). In this also by the minor herself in order to be effective as an
case, however, it is not so situated, for according to him he express pardon under Art. 344 of the Revised Penal Code.
was at the plant of the Republic Asahi Glass Corporation in Thus, in the case of People vs. Lacson, Jr., (C.A.) 55 O.G.
Barangay Pinagbuhatan, Pasig, which is but a few 9460, we find the following words: Neither must we be
kilometers from Barangay Sumilang of the same understood as supporting the view that the parents alone can
municipality where the crime was committed. extend a valid pardon. Far from it, for we, too are of the
belief that the pardon by the parents, standing alone, is
Rape; Accused-appellant tries to discredit the victims inefficacious. It was also held in another case, that The
testimony by questioning her behavior after she was express pardon of a person guilty of attempted abduction of
allegedly raped. The court ruled that it is not proper to judge a minor, granted by the latters parents, is not sufficient to
the actions of children who have undergone traumatic remove criminal responsibility, but must be accompanied by
experience by the norms of behavior expected under the the express pardon of the girl herself. (U.S. vs. Luna, 1 Phil.
circumstances from mature people. The range of emotion 360).
shown by rape victims is yet to be captured even by the
calculus. It is thus unrealistic to expect uniform reactions
from rape victims.It should be borne in mind, in this FACTS:
connection, that the victim was only a naive nine (9) year old Complainant Estela Santos owns a house at No. 6 Dr. Garcia
child when the crime was committed on her. She considered St., in Barangay Sumilang, Pasig, she resides with her
common-law husband and their minor daughter, Maristel
Cruz. Behind the said house, complainant also owns a three- In 1992, Rodrigo Cawili borrowed various sums of money
door apartment building, one unit of which was rented and from petitioner Panaguiton, Jr. On January 8, 1993, Cawili
occupied by accused Irvin Tadulan, his wife Adefa Tadulan and Ramon C. Tongson (Tongson) jointly issued in favor of
and their three children name Dianne, Angie and Bochoy Panaguiton, Jr. three (3) checks in payment of the said loans.
who were aged 10, 9 and 5, respectively. In 1992 Upon presentment for payment on March 18, 1993, the
complainants daughter, Maristel Cruz was about nine (9) checks were dishonored.
year old and was in grade school. She often played with the On August 24, 1995, Panaguiton, Jr. filed a complaint against
accuseds children in the vicinity of their house and the Cawili and Tongson for violating Batas Pambansa (B.P.)
apartment building. Bilang 22 before the Quezon City Prosecutor's Office.
However, on March 15, 1999, Assistant City Prosecutor
That on or about the 2nd day of April, 1992 in the Sampaga held that the case had already prescribed pursuant
Municipality of Pasig, Metro Manila, Philippines and within to Act No. 3326, as amended, which provides that violations
the jurisdiction of this Honorable Court, armed with a knife, penalized by B.P. Blg. 22 shall prescribe after four (4) years
with lewd design and by means of force, threats and starting on the date the checks were dishonored (March 18,
intimidation, did then and there willfully, unlawfully and 1993) Panaguiton, Jr. appealed to the DOJ but it dismissed
feloniously have sexual intercourse with one Maristel Cruz, a the case on the same grounds of prescription.
minor, nine (9) years old, without her consent and against The DOJ explained that Act No. 3326 applies to violations of
her will. special acts that do not provide for a prescriptive period for
the offenses thereunder. Since B.P. Blg. 22, as a special act,
Estela Santos immediately informed the wife of Irvin does not provide for the prescription of the offense it defines
Tadulan that her husband has raped her daughter. She and punishes, Act No. 3326 applies to it, and not Art. 90 of
further informed Adefa Tadulan that she would not take the Revised Penal Code which governs the prescription of
action against the latters husband if they would vacate the offenses penalized thereunder. The DOJ also cited the case
apartment unit right away. Adefa Tadulan later on met with of Zaldivia v. Reyes, Jr., wherein the SC ruled that the
Estela Santos and told her that she had driven away Irvin proceedings referred to in Act No. 3326, as amended, are
Tadulan, but requested that she and her children be allowed judicial proceedings, and not the one before the prosecutor's
to stay until Saturday, April 11, 1992. Estela Santos thereafter office (administrative proceedings). Therefore, the filing of a
noted, however, that Irvin Tadulan was still coming home to complaint with the Office of the City Prosecutor of Quezon
the apartment unit every night despite the promise of his City does not interrupt the running of the prescriptive period
wife that she herself would call the police should he ever for violation of B.P. Blg. 22. Hence, this Petition for Review.
come back to the place.
Issues:
ISSUE: Whether or not the offense of violation of B.P. Blg. 22 has
Whether or not the court erred in disregarding the defense of already prescribed per Act No. 3326
pardon and alibi of the accused?
PANAGUITON, JR. v. DOJ Ruling:
Facts: NO.
As correctly stated by the DOJ, Act No. 3326 is the law Laws and Municipal Ordinances), which provides that
applicable to offenses under special laws which do not prescription shall begin to run from the day of the
provide for their own prescriptive periods. However, as ruled commission of the violation of the law, and if the same not be
in the cases of Ingco v. Sandiganbayan, Sanrio known at the time, from the discovery thereof and the
Company Limited v. Lim, and Securities and institution of judicial proceedings for its investigation and
Exchange Commission v. Interport Resources punishment. overwhelm
Corporation, et al, the Court ruled that the prescriptive
period is interrupted by the institution of proceedings for
preliminary investigation against the accused. Although The prescription shall be interrupted when proceedings are
Panaguiton, Jr. went through the proper channels, within the instituted against the guilty person, and shall begin to run
prescribed periods, an aggregate period of nine (9) years had again if the proceedings are dismissed for reasons not
elapsed from the time he filed his complaint-affidavit up to constituting jeopardy.
the time the DOJ issued the assailed resolution. Aggrieved
parties, especially those who do not sleep on their rights and In cases involving violations of R.A. No. 3019 committed
actively pursue their causes, should not be allowed to suffer prior to the February 1986 Edsa Revolution that ousted
unnecessarily further simply because of circumstances President Ferdinand E. Marcos, we ruled that the
beyond their control. Notwithstanding whether or not the government as the aggrieved party could not have known of
proceeding is a judicial proceeding or an administrative the violations at the time the questioned transactions were
proceeding, Panaguiton, Jr.s filing of the complaint
made (PCGG vs. Desierto, G.R. No. 140232, January 19,
interrupted the running of the prescriptive period.
2001, 349 SCRA 767; Domingo v. Sandiganbayan, supra,
Note 14; Presidential Ad Hoc Fact Finding Committee on
Behest Loans v. Desierto, supra, Note 16). Moreover, no
person would have dared to question the legality of those
transactions. Thus, the counting of the prescriptive period
commenced from the date of discovery of the offense in 1992
PRESIDENTIAL COMMISSION ON GOOD after an exhaustive investigation by the Presidential Ad Hoc
GOVERNMENT (PCGG) vs. HON. ANIANO A. Committee on Behest Loans.-Presidential Ad Hoc Fact-
DESIERTO, ET AL. Finding Committee on Behest Loans v. Desierto (2001)

Prescriptive period for Anti-Graft and Corrupt


Practices Act (RA No. 3019, as amended) RULING: Respondent Ombudsman committed grave abuse
of discretion in dismissing the subject complaint on the
ground of prescription.
The applicable law in the computation of the prescriptive
period is Section 2 of Act No. 3326, as amended (Act
Establishing Prescriptive Periods for Violations of Special Respondents members of the PNB Board of Directors and
Officers of NOCOSII are charged with violation of R.A. No.
3019, a special law. Amending said law, Section 4, Batas the time of discovery. Thus, the filing of the complaint was
Pambansa Blg. 195, increased the prescriptive period from well within the prescriptive period of 15 years.
ten to fifteen years.
FACTS: On October 8, 1992, President Fidel V. Ramos
The issue of prescription has long been laid to rest in the issued Administrative Order No. 13 creating the Presidential
aforementioned Presidential Ad Hoc Fact-Finding Ad Hoc Fact-Finding Committee on Behest Loans
Committee on Behest Loans v. Desierto, where the Court (Committee) which was tasked to inventory all behest loans,
held that it was well-nigh impossible for the State, the determine the parties involved and recommend whatever
aggrieved party, to have known the violations of R.A. No. appropriate actions to be pursued thereby and Memorandum
3019 at the time the questioned transactions were made Order No. 61 expanded the functions of the Committee to
because, as alleged, the public officials concerned connived include the inventory and review of all non-performing
or conspired with the "beneficiaries of the loans. Thus, we loans, whether behest or non-behest.
agree with the COMMITTEE that the prescriptive period for
the offenses with which respondents in OMB-0-96-0968 Among the accounts referred to the Committee's Technical
were charged should be computed from the discovery of the Working Group (TWG) were the loan transactions between
commission thereof and not from the day of such Northern Cotabato Sugar Industries, Inc. (NOCOSII) and
commission. Philippine National Bank (PNB) and the Committee
classified the loans obtained by NOCOSII from PNB as
The assertion by the Ombudsman that the phrase 'if the behest because of NOCOSII's insufficient capital and
same not be known' in Section 2 of Act No. 3326 does not inadequate collaterals after it had examined and studied all
mean 'lack of knowledge' but that the crime 'is not the documents relative to the said loan transactions.
reasonably knowable' is unacceptable, as it provides an
interpretation that defeats or negates the intent of the law, Based on the Sworn Statement of PCGG consultant Orlando
which is written in a clear and unambiguous language and Salvador, petitioner filed with the Office of the Ombudsman
thus provides no room for interpretation but only the criminal complaint against respondents on April 5, 1995.
application. Petitioner alleges that respondents violated the following
provisions of Section 3 (e) and (g) of R.A. No. 3019.
As to when the period of prescription was interrupted, the
second paragraph of Section 2, Act No. 3326, as amended, The respondents failed to submit any responsive pleading
provides that prescription is interrupted 'when proceedings before the Ombudsman, prompting Graft Investigator
are instituted against the guilty person. Officer (GIO) I Melinda S. Diaz-Salcedo to resolve the case
based on the available evidence. In a Resolution, GIO Diaz-
Records show that the act complained of was discovered in Salcedo recommended the dismissal of the case on the
1992. The complaint was filed with the Office of the ground of insufficiency of evidence or lack of probable cause
Ombudsman on April 5, 1995, or within three (3) years from against the respondents and for prescription of the offense.
Ombudsman Desierto approved the recommendation. information in court by the state prosecutors
Petitioner filed a Motion for Reconsideration but it was because up to that time the original trust
denied by GIO Diaz-Salcedo, which was approved by relation may be converted by the parties into
Ombudsman Desierto. an ordinary creditor-debtor situation,
thereby placing the complainant in estoppel
to insist on the original trust. But after the
ISSUE: Whether or not the Ombudsman committed grave
justice authorities have taken cognizance of
abuse of discretion in ruling that the offense leveled against the crime and instituted action in court, the
respondents has prescribed. offended party may no longer divest the
prosecution of its power to exact the
criminal liability, as distinguished from the
SOCIAL SECURITY SYSTEM G.R. civil. The crime being an offense against the
No. 158131 state, only the latter can renounce it.
vs It may be observed in this regard
DEPARTMENT OF JUSTICE that novation is not one of the means
August 8, 2007 recognized by the Penal Code whereby
criminal liability can be extinguished;
RULING hence, the role of novation may only be to
The Supreme Court ruled that SENCORs criminal liability either prevent the rise of criminal liability or
was not extinguished. The SC argued that for novation to to cast doubt on the true nature of the
apply there must be an original contract to speak of. In this original basic transaction, whether or not it
case, the novation does not apply because there was no was such that its breach would not give rise
original contract that can be replaced by a new contract to penal responsibility, as when money
changing the object or principal condition of the original loaned is made to appear as a deposit, or
contract, substituting the person of the debtor, or other similar disguise is resorted to.
subrogating a third person in the rights of the creditor.

The Court observed that although novation is not one of the FACTS
means recognized by the Revised Penal Code to extinguish respondent Martels are directors of SENCOR, an
criminal liability, it may prevent the rise of criminal liability information technology firm. Petitioner is a government-
or to cast doubt on the true nature of the original basic owned and controlled corporation mandated by its charter,
transaction, provided the novation takes place before the RA 1161, to provide financial benefits to private sector
filing of the Information with the trial court. In the case employees. Petitioner filed with the Pasay City Prosecutors
People v. Nery, We held: Office a complaint against Martels and their five co-accused
for SENCORs non-payment of contributions. To pay this
The novation theory may perhaps amount, respondent Martels offered to assign to petitioner a
apply prior to the filing of the criminal parcel of land in Tagaytay City. Petitioner accepted the offer
subject to the condition that respondent Martels will settle only ten (10) years. Thus, for offenses allegedly committed by
their obligation either by way of dacion en pago or through the petitioner from 1962 up to March 15, 1982, the same
cash settlement within a reasonable time. Thus, petitioner shall prescribe in 10 years. On the other hand, for offenses
withdrew its complaint but reserved its right to revive the allegedly committed by the petitioner during the period from
same in the event that no settlement is arrived at. In March 16, 1982 until 1985, the same shall prescribe in 15
December 2001, respondent Jose V. Martel wrote petitioner years.
offering, in lieu of the Tagaytay City property, computer-
related services. Petitioner filed with the Pasay City As to when these two periods begin to run, reference is made
Prosecutors Office another complaint against respondent to Act No. 3326 which governs the computation of
Martels for SENCORs non-remittance of contributions. prescription of offenses defined by and penalized under
special laws. Thus, this Court rules that the prescriptive
ISSUE period of the offenses herein began to run from the discovery
Whether or not the concept of novation serves to abate the thereof or on May 8, 1987, which is the date of the complaint
criminal liability of the respondents filed by the former Solicitor General Francisco I. Chavez
against the petitioner with the PCGG.
Part I, Case No. 70
Therefore, when the Office of the Special Prosecutor initiated
BENJAMIN ("KOKOY") T. ROMUALDEZ, petitioner, the preliminary investigation of Criminal Case Nos. 13406-
vs.HON. SIMEON V. MARCELO, in his official 13429 on March 3, 2004 by requiring the petitioner to
capacity as the Ombudsman, and PRESIDENTIAL submit his counter-affidavit, the alleged offenses subject
COMMISSION ON GOOD GOVERNMENT, therein have already prescribed.
respondents.
G.R. Nos. 165510-33, July 28, 2006 2 No. Under Section 2 of Act No. 3326, the prescriptive period
shall be interrupted when proceedings are instituted against
Intro/Doctrine/Ruling: the guilty person. However, there is no such proceeding
instituted against the petitioner to warrant the tolling of the
Sec. 7 and 11 of RA 3019; Sec. 2 of Act No. 3326; prescriptive periods of the offenses charged against him. In
Article 91 of the RPC cannot be applied suppletorily to Act Romualdez v. Sandiganbayan, petitioner averred that PCGG
No. 3326 because such Act is not a special law within the acted without jurisdiction and/or grave abuse of discretion
ambit of Article 10 of the RPC. in conducting a preliminary investigation of cases not falling
within its competence. This Court, in its resolve to deal with
the merits of the case to remove the possibility of any
1 Yes. Section 11 of RA No. 3019 provides that all offenses misunderstanding as to the course which it wishes
punishable therein shall prescribe in 15 years. But prior to petitioners cases in the Sandiganbayan to take declared
the amendment of Section 11 of R.A. No. 3019 by B.P. Blg. invalid the preliminary investigation conducted by the PCGG
195 which was approved on March 16, 1982, the prescriptive over the 24 offenses ascribed to Romualdez.
period for offenses punishable under the said statute was
In Romualdez v. Sandiganbayan, petitioner assailed the makes the Code suppletory to special laws, Act No. 3326
validity of the informations filed with the Sandiganbayan in cannot fall within the ambit of special law as contemplated
Criminal Case Nos. 13406-13429 considering that the same and used in Article 10 of the RPC because it does not define
were subscribed and filed by the PCGG. In granting any acts which are punishable and provides penalties for
petitioners plea, this Court held, thus: them. Special law is defined as penal laws that punish acts
not defined and penalized by the Penal Code of the
Here, the informations were filed by an Philippines.
unauthorized party. The defect cannot be
cured by conducting another preliminary
investigation. An invalid information is no Thus, Art. 91 cannot apply suppletorily to Act No. 3326.
information at all and cannot be the basis for
criminal proceedings. Facts: Petitioner is being charged with violations of Section
7 of RA No. 3019 for failure to file his Statements of Assets
Indeed, the nullity of the proceedings initiated by then and Liabilities for the period 1967-1985 during his tenure as
Solicitor General Chavez in 1987 with the PCGG and by the Ambassador Extraordinary and Plenipotentiary and for the
PCGG with the Sandiganbayan in 1989 is judicially settled. period 1963-1966 during his tenure as Technical Assistant in
In contemplation of the law, no proceedings exist that could the DFA. He claims that the court should dismiss the
have merited the suspension of the prescriptive periods. criminal cases pending against him on the ground of
prescription.
3 No. Section 2 of Act No. 3326 provides that the prescription
shall begin to run from the day of the commission of the The Ombudsman and the PCGG argue that that the filing of
violation of the law, and if the same be not known at the the complaint with the Presidential Commission on Good
time, from the discovery thereof and the institution of Government (PCGG) in 1987 and the filing of the
judicial proceedings for its investigation and punishment. information with the Sandiganbayan in 1989 interrupted the
The running of the prescriptive period shall be interrupted prescriptive period; that the absence of the petitioner from
when proceedings are instituted against the guilty person, the Philippines from 1986 until 2000 also interrupted the
and shall begin to run again if the proceedings are dismissed aforesaid period based on Article 91 of the Revised Penal
for reasons not constituting jeopardy. Clearly, Section 2 of Code; that considering that both RA No. 3019 and Act No.
Act No. 3326 did not provide that the absence of the accused 3326 or the Act To Establish Periods of Prescription For
from the Philippines prevents the running of the prescriptive Violations Penalized By Special Acts and Municipal
period. Thus, the only inference that can be gathered from Ordinances and to Provide When Prescription Shall Begin
the foregoing is that the legislature, in enacting Act No. To Run, are silent as to whether prescription should begin to
3326, did not consider the absence of the accused from the run when the offender is absent from the Philippines, the
Philippines as a hindrance to the running of the prescriptive Revised Penal Code, which answers the same in the negative,
period. Expressio unius est exclusio alterius. should be applied.

Also, while it is true that Article 10 of the Revised Penal Code Issues: 1. Whether the offenses for which petitioner are
being charged have already prescribed? Article 248. Murder. - Any person who, not falling within
2 WON the filing of the complaint with the PCGG and the provisions of Article 246, shall kill another, shall be
Sandiganbayan interrupted the running of the prescriptive guilty of murder and shall be punished by reclusion perpetua
period? to death if committed with any of the following attendant
3 WON Article 91 of the RPC applies suppletorily to Act No. circumstances:
3326, such that it suspended the running of the prescriptive
period of 10 or 15 years in RA 3019? 1) With treachery, taking advantage of superior strength,
with the aid of armed men, or employing means to weaken
Case #72. the defense or of means or persons to insure or afford
People of the Philippines vs Vergara impunity.
GR No 177763; July 3, 2013
Doctrines: Credibility of Witnesses:
Self-Defense, elements: (1) unlawful aggression on the When it comes to the matter of credibility of a witness,
part of the victim; (2) reasonable necessity of the means settled are the guiding rules some of which are that (1) the
employed to prevent or repel such aggression; and (3) lack of Appellate court will not disturb the factual findings of the
sufficient provocation on the part of the person resorting to lower Court, unless there is a showing that it had overlooked,
self-defense. misunderstood or misapplied some fact or circumstance of
A person who invokes self-defense has the burden of proof. weight and substance that would have affected the result of
He must prove all the elements of self-defense. However, the the case, which showing is absent herein; (2) the findings of
most important of all the elements is unlawful aggression on the Trial Court pertaining to the credibility of a witness is
the part of the victim. entitled to great respect since it had the opportunity to
Unlawful aggression must be proved first in order for examine his demeanor as he testified on the witness stand,
self-defense to be successfully pleaded, whether complete or and, therefore, can discern if such witness is telling the truth
incomplete. Unlawful aggression is an actual physical or not; and (3) a witness who testifies in a categorical,
assault, or at least a threat to inflict real imminent injury, straightforward, spontaneous and frank manner and remains
upon a person. In case of threat, it must be offensive and consistent on cross-examination is a credible witness.
strong, positively showing the wrongful intent to cause
injury. It "presupposes actual, sudden, unexpected or Facts: Accused Vergara and Inocencio were charged of
imminent danger - not merely threatening and intimidating murder qualified by treachery. Victim was allegedly stabbed
action." It is present "only when the one attacked faces real by the accused while the latter were causing some
and immediate threat to ones life." disturbance in the street. Exchange of words lead to the
Treachery, how manifested: the offender commits any incident initiated by the accused and the retort of the victim
of the crimes against persons, employing means, methods, or was not likened by the former.
forms in the execution, which tend directly and specially to Issues:
insure its execution, without risk to the offender arising from 1. Whether or not the accused had basis to
the defense which the offended party might make. interpose self-defense? No, the accused had no basis to
interpose self-defense. Elements are not all present. In fact,
the unlawful aggression came from the accused and not the There is also evident premeditation because the evidence
victim. The victim was passing in peace when they shows that a couple of days before the actual shooting of
approached him and initiated the act of killing causing the Atty. Alipio, Raymundo Zamora already saw and heard
life of the victim in peril. accused-appellants Norberto (Jun) Adviento, Renato Ramos,
2. Whether or not treachery is present? Yes, treachery and Lolito Aquino, talking to Francisca Talaro and coming to
is present. Here, accused-appellant Vergara after exchanging an agreement to kill Atty. Alipio.
words with the victim, threw his arm around the victims Pitted against the prosecution evidence, accused-appellants'
shoulder and proceeded to stab him. The victim was totally only defense is that the evidence is insufficient to prove they
unaware of the evil that would befall him. The number and are part of the conspiracy to commit the murder. Said
severity of the wounds received by the victim indicated that defense is sorely wanting when pitted against the
he was rendered immobile and without any real opportunity prosecution evidence.
to defend himself other than feebly raising his arm to ward In People v. Bautista, the Court reiterated the hornbook
off the attack. principle of conspiracy, to wit:
Conspiracy exists when two or more persons come to an
PEOPLE OF THE PHILIPPINES VS. TALARO ET. AL agreement concerning the commission of a felony and decide
G.R. No. 175781; March 20, 2012 to commit it. Where all the accused acted in concert at the
time of the commission of the offense, and it is shown by
Doctrine: such acts that they had the same purpose or common design
Article 248 of the Revised Penal Code is defined as the and were united in its execution, conspiracy is sufficiently
unlawful killing of a person, which is not parricide or established. It must be shown that all participants performed
infanticide, attended by circumstances such as treachery or specific acts which such closeness and coordination as to
evident premeditation. The presence of any one of the indicate a common purpose or design to commit the felony.
circumstances enumerated in Article 248 of the Code is Each conspirator is responsible for everything done by his
sufficient to qualify a killing as murder. confederates which follows incidentally in the execution of a
In People v. Sanchez, the Court held that the essence of common design as one of its probable and natural
treachery is the sudden attack by an aggressor without the consequences even though it was not intended as part of the
slightest provocation on the part of the victim, depriving the original design. x x x (Emphasis supplied)
latter of any real chance to defend himself, thereby ensuring In this case, the existence of a conspiracy has been
the commission of the crime without risk to the aggressor. established by the testimony of Raymundo Zamora,
There can be no cavil that the evidence on record shows positively identifying all three accused-appellants as the ones
treachery in the killing of Atty. Alipio, thus qualifying the he saw and heard transacting with Francisca Talaro on April
crime as murder. The assailant, identified as accused- 24, 1994 to kill Atty. Melvin Alipio for the price of
appellant Renato Ramos, just suddenly fired upon Atty. P60,000.00, and pointing to Lolito Aquino as the one who
Alipio at a very close distance, without any provocation from demanded and received part of the payment after Atty. Alipio
said unarmed victim, who was then just conversing with had been killed.
some other people.
Facts:
Raymundo Zamora is the nephew of Gregorio Talaro, the Issue: Whether or not there was treachery, evident
husband of Francisca Talaro. In the morning of April 24, premeditation and conspiracy in the given case.
1994, when Zamora went home for breakfast after driving his
tricycle, he found Francisca Talaro, Lolito Aquino, Renato
Atong Ramos, and Norberto Jun Adviento conversing among DAYAP VS SENDIONG
themselves. He heard that Francisca Talaro, was transacting
with the other three accused-appellants for the killing of Article 365 of the Revised Penal Code punishes any
Atty. Melvin Alipio in consideration for a certain amount. person who, by reckless imprudence, commits any act which,
The three accused-appellants then nodded their heads in had it been intentional, would constitute a grave felony, with
agreement. After learning of the group's plan, Zamora got the penalty of arresto mayor in its maximum period to
scared and stayed away from the group, but three days after prision correccional in its medium period. When such
that meeting in front of his house, he was asked by Francisca reckless imprudence the use of a motor vehicle, resulting in
Talaro to drive her and her husband Gregorio to Brgy. the death of a person attended the same article imposes upon
Bactad. The Talaro spouses alighted at a place in Brgy. the defendant the penalty of prision correccional in its
Bactad, while Zamora stayed in his tricycle and merely medium and maximum periods;
waited for them. He assumed that the couple delivered the
payment to someone in Brgy. Bactad. Where a reckless, imprudent, or negligent act
Another eyewitness, Eusebio Hidalgo, whose son was results in two or more grave or less grave felonies, a
confined at the clinic, was sitting at a bench in the garage of complex crime is committed. Article 48 of the Revised
the clinic on the morning of April 26, 1994. Two other Penal Code provides that when the single act constitutes two
women who were looking for Atty. Alipio also sat at the or more grave or less grave felonies, or when an offense is a
bench with him after he told them that Atty. Alipio was still necessary means for committing the other, the penalty for
having his breakfast. After a few minutes, a man arrived the most serious crime shall be imposed, the same to be
looking for Dr. Alipio, and also sat at the bench. Thereafter, applied in its maximum period. Since Article 48 speaks of
Atty. Alipio came out to the garage and talked to the two felonies, it is applicable to crimes through negligence in view
women. When Atty. Alipio finished talking to them, the man of the definition of felonies in Article 3 as acts or omissions
sitting with them on the bench suddenly stood up and shot punishable by law committed either by means of deceit
Atty. Alipio three times. Atty. Alipio was merely one meter (dolo) or fault (culpa). Thus, the penalty imposable upon
away from the assailant when the latter shot him. After the petitioner, were he to be found guilty, is prision correccional
shooting, the assailant walked away. Hidalgo then saw the in its medium period (2 years, 4 months and 1 day to 4 years)
helper at the clinic, Reny Balanga, run after the assailant, but and maximum period (4 years, 2 months and 1 day to 6
the latter had whistled to his companion who was waiting on years).
his motorcycle and the two were able to speed away aboard
said vehicle. Hidalgo identified the assailant from a picture FACTS:
shown to him.The picture was that of Renato Ramos. Dayap allegedly drove in a reckless manner a 10-wheeler
cargo truck hitting an automobile driven by Sendiong who
was with two female passengers. Such incident caused the
death of Sendiong, less serious physical injuries on the
bodies of the two female passengers and extensive damage to Article 267 of the RPC
the automobile. Hence, Dayap was charged with the crime of
Reckless Imprudence resulting to Homicide, Less Serious Facts:
Physical Injuries, and Damage to Property.
Christopher, a two-year old minor, had disappeared from
ISSUE: mcdonalds in Binondo when his mom and his sister were
WON the RTC has jurisdiction to hear a criminal case ordering from the counter. On February 25, 2001, Teresa
involving complex crimes such as reckless imprudence received a call from a woman and claimed to have custody of
resulting in homicide, less serious physical injuries and Christopher and asked forP30,000 in exchange for the boy.
damage to property? They then agreed to conduct the pay off in the morning of
April 7, 2001 at Pitangs Carinderia in Kapatagan, Lanao
RULING: del Norte.
NO. When this case was filed on 29 December 2004, Section
32(2) of Batas Pambansa Bilang 129 had already been Issue:
amended by R.A. No. 7691. R.A. No. 7691 extended the Whether the two accused are guilty of violating the crime of
jurisdiction of the first-level courts over criminal cases to Kidnapping for Ransom under Article 267 ofthe RPC, as
include all offenses punishable with imprisonment not amended by RA No. 7659?
exceeding six (6) years irrespective of the amount of fine,
and regardless of other imposable accessory or other Ruling: Yes
penalties including those for civil liability. It explicitly states Kidnapping; Elements.The crime has the following
"that in offenses involving damage to property through elements: (1) the offender is a private individual; not either
criminal negligence, they shall have exclusive original of the parents of the victim or a public officer who has a duty
jurisdiction thereof." It follows that criminal cases for under the law to detain a person; (2) he kidnaps or detains
reckless imprudence punishable with prision another, or in any manner deprives the latter of his liberty;
correccional in its medium and maximum periods (3) the act of detention or kidnapping must be illegal and (4)
should fall within the jurisdiction of the MTC and in the commission of the offense, any of the following
not the RTC. Clearly, therefore, jurisdiction to hear and try circumstances is present: (a) the kidnapping or detention
the same pertained to the MTC and the RTC did not have lasts for more than three days; (b) it is committed by
original jurisdiction over the criminal case. Consequently, simulating public authority; (c) any serious physical injuries
the MTC of Sibulan, Negros Oriental had properly taken are inflicted upon the person kidnapped or detained or
cognizance of the case and the proceedings before it were threats to kill him are made or (d) the person kidnapped or
valid and legal. detained is a minor, female or a public official.
The essence of the crime of kidnapping is the actual
People vs Mamantak deprivation of the victims liberty coupled with the intent of
GR no. 174659 the accused to effect it; If the victim is a minor, or the victim
July 28, 2008 is kidnapped and illegally detained for the purpose of
extorting ransom, the duration of his detention becomes Pajarillo shot Evangeline with his service shotgun hitting her
inconsequential. in the abdomen instantly causing her death.
Lauro Tangco, Evangeline's husband, together with his six
minor children (respondents) filed with the Regional Trial
SAFEGUARD SECURITY vs. TANGCO Court (RTC) of Quezon City, a criminal case of Homicide
GR No. 165732, December 14, 2006 against Pajarillo, docketed as Criminal Case No. 0-97-73806
Austria Martinez, J.: and assigned to Branch 78. Respondents reserved their right
to file a separate civil action in the said criminal case. The
DOCTRINE: An act or omission causing damage to another RTC of Quezon City subsequently convicted Pajarillo of
may give rise to two separate civil liabilities on the part of the Homicide in its Decision dated January 19, 2000. On appeal
offender, i.e., (1) civil liability ex delicto, under Article 100 of to the CA, the RTC decision was affirmed with modification
the Revised Penal Code; and (2) independent civil liabilities, as to the penalty in a Decision.
such as those (a) not arising from an act or omission Meanwhile, on January 14, 1998, respondents filed with
complained of as a felony, e.g., culpa contractual or RTC, Branch 273, Marikina City, a complaint5 for damages
obligations arising from law under Article 31 of the Civil against Pajarillo for negligently shooting Evangeline and
Code, intentional torts under Articles 32 and 34, and culpa against Safeguard for failing to observe the diligence of a
aquiliana under Article 2176 of the Civil Code; or (b) where good father of a family to prevent the damage committed by
the injured party is granted a right to file an action its security guard. Respondents prayed for actual, moral and
independent and distinct from the criminal action under exemplary damages and attorney's fees.
Article 33 of the Civil Code. Either of these liabilities may be In their Answer,6 petitioners denied the material allegations
enforced against the offender subject to the caveat under in the complaint and alleged that Safeguard exercised the
Article 2177 of the Civil Code that the offended party cannot diligence of a good father of a family in the selection and
recover damages twice for the same act or omission or under supervision of Pajarillo; that Evangeline's death was not due
both causes. to Pajarillo's negligence as the latter acted only in self-
Provisions mentioned: Art. 2176, 2177 of the Civil defense.
code, Article 103 of the RPC The RTC found respondents to be entitled to damages. It
FACTS: rejected Pajarillo's claim that he merely acted in self-defense.
On November 3, 1997, at about 2:50 p.m., Evangeline It gave no credence to Pajarillo's bare claim that Evangeline
Tangco (Evangeline) went to Ecology Bank, Katipunan was seen roaming around the area prior to the shooting
Branch, Quezon City, to renew her time deposit per advise of incident since Pajarillo had not made such report to the head
the bank's cashier as she would sign a specimen card. office and the police authorities. The RTC further ruled that
Evangeline, a duly licensed firearm holder with being the guard on duty, the situation demanded that he
corresponding permit to carry the same outside her should have exercised proper prudence and necessary care
residence, approached security guard Pajarillo, who was by asking Evangeline for him to ascertain the matter instead
stationed outside the bank, and pulled out her firearm from of shooting her instantly; that Pajarillo had already been
her bag to deposit the same for safekeeping. Suddenly, convicted of Homicide in Criminal Case No. 0-97-73806;
and that he also failed to proffer proof negating liability in extinguished with the extinction of the penal liability with a
the instant case. pronouncement that the fact from which the civil action
might proceed does not exist; that unlike in civil liability
The RTC also found Safeguard as employer of Pajarillo to be arising from quasi-delict, the defense of diligence of a good
jointly and severally liable with Pajarillo. It ruled that while father of a family in the employment and supervision of
it may be conceded that Safeguard had perhaps exercised employees is inapplicable and irrelevant in civil liabilities
care in the selection of its employees, particularly of based on crimes or ex-delicto; that Article 103 of the Revised
Pajarillo, there was no sufficient evidence to show that Penal Code provides that the liability of an employer for the
Safeguard exercised the diligence of a good father of a family civil liability of their employees is only subsidiary, not joint
in the supervision of its employee; that Safeguard's evidence or solidary.
simply showed that it required its guards to attend trainings ISSUE/S:
and seminars which is not the supervision contemplated Whether or not the CA erred in ruling that the liability of
under the law; that supervision includes not only the Safeguard Security is subsidiary
issuance of regulations and instructions designed for the HELD:
protection of persons and property, for the guidance of their YES. Safeguard insists that the claim for damages by
servants and employees, but also the duty to see to it that respondents is based on culpa aquiliana under Article 217611
such regulations and instructions are faithfully complied of the Civil Code, in which case, its liability is jointly and
with. severally with Pajarillo. However, since it has established
Petitioners appealed the RTC decision to the CA. On July 16, that it had exercised due diligence in the selection and
2004, the CA issued its assailed Decision, the dispositive supervision of Pajarillo, it should be exonerated from civil
portion of which reads: liability.
IN VIEW OF ALL THE FOREGOING, the appealed decision The law at the time the complaint for damages was filed is
is hereby AFFIRMED, with the modification that Safeguard Rule 111 of the 1985 Rules on Criminal Procedure, as
Security Agency, Inc.'s civil liability in this case is only amended, to wit:
subsidiary under Art. 103 of the Revised Penal Code.
In finding that Safeguard is only subsidiarily liable, the CA SECTION 1. Institution of criminal and civil
held that the applicable provisions are not Article 2180 in actions. - When a criminal action is instituted, the
relation to Article 2176 of the Civil Code, on quasi-delicts, civil action for the recovery of civil liability is
but the provisions on civil liability arising from felonies impliedly instituted with the criminal action, unless
under the Revised Penal Code; that since Pajarillo had been the offended party waives the civil action, reserves
found guilty of Homicide in a final and executory judgment his right to institute it separately, or institutes the
and is said to be serving sentence in Muntinlupa, he must be civil action prior to the criminal action.
adjudged civilly liable under the provisions of Article 100 of Such civil action includes recovery of indemnity
the Revised Penal Code since the civil liability recoverable in under the Revised Penal Code, and damages under
the criminal action is one solely dependent upon conviction, Articles 32, 33, 34, and 2176 of the Civil Code of the
because said liability arises from the offense charged and no Philippines arising from the same act or omission
other; that this is also the civil liability that is deemed of the accused.
Respondents reserved the right to file a separate civil action and found guilty or acquitted, provided that the offended
and in fact filed the same on January 14, 1998. party is not allowed, if he is actually charged also criminally,
The CA found that the source of damages in the instant case to recover damages on both scores, and would be entitled in
must be the crime of homicide, for which he had already such eventuality only to the bigger award of the two,
been found guilty of and serving sentence thereof, thus must assuming the awards made in the two cases vary. In other
be governed by the Revised Penal Code words, the extinction of civil liability referred to in Par. (e) of
An act or omission causing damage to another may give rise Section 3, Rule 111, refers exclusively to civil liability founded
to two separate civil liabilities on the part of the offender, on Article 100 of the Revised Penal Code, whereas the civil
i.e., (1) civil liability ex delicto, under Article 100 of the liability for the same act considered as quasi-delict only and
Revised Penal Code; and (2) independent civil liabilities, not as a crime is not extinguished even by a declaration in
such as those (a) not arising from an act or omission the criminal case that the criminal act charged has not
complained of as a felony, e.g., culpa contractual or happened or has not been committed by the accused. Briefly
obligations arising from law under Article 31 of the Civil stated, We here hold, in reiteration of Garcia, that culpa
Code, intentional torts under Articles 32 and 34, and culpa aquiliana includes voluntary and negligent acts which may
aquiliana under Article 2176 of the Civil Code; or (b) where be punishable by law.
the injured party is granted a right to file an action The civil action filed by respondents was not derived from
independent and distinct from the criminal action under the criminal liability of Pajarillo in the criminal case but one
Article 33 of the Civil Code. Either of these liabilities may be based on culpa aquiliana or quasi-delict which is separate
enforced against the offender subject to the caveat under and distinct from the civil liability arising from crime. The
Article 2177 of the Civil Code that the offended party cannot source of the obligation sought to be enforced in the civil
recover damages twice for the same act or omission or under case is a quasi-delict not an act or omission punishable by
both causes. law.
A reading of respondents' complaint shows that the latter are Since the civil liability was founded on a quasi-delict and not
invoking their right to recover damages against Safeguard for from the crime of homicide. The liability of Safeguard
their vicarious responsibility for the injury caused by Security must be Solidary and not Subsidiary.
Pajarillo's act of shooting and killing Evangeline under
Article 2176, Civil Code. The scope of Article 2176 is not
limited to acts or omissions resulting from negligence. Well- G.R. No. 178115 July 28, 2014
entrenched is the doctrine that Article 2176 covers not only
acts committed with negligence, but also acts which are
PEOPLE OF THE PHILIPPINES, Plaintiff-Appellee,
voluntary and intentional.
In Elcano vs Hill, the court ruled that: Article 2176, where it vs.
refers to "fault or negligence," covers not only acts "not JOJO SUMILHIG, RICARDO SUMILHIG alias
punishable by law" but also acts criminal in character, CARDING SUMILHIG, PASOT SALOLI, ERIC ENOC,
whether intentional and voluntary or negligent. WARLITO MONTEBON,* and CIO
Consequently, a separate civil action lies against the offender LIMAMA, Accused,
in a criminal act, whether or not he is criminally prosecuted
JOJO SUMILHIG, RICARDO SUMILHIG alias Ruling of the Regional Trial Court
CARDING SUMILHIG, and PASOT
SALOLI, Accused-Appellants. The RTC convicted the appellants ofthe complex crime of
double murder and double frustrated murder and sentenced
them to suffer the penalty of death. It further ordered them
to indemnify, jointlyand severally, the heirs of Cresjoy and
FACTS: Appellants, together with the accused Eric Enoc, Rolly the sum of P100,000.00 as civil indemnity, and the
Warlito Montibon and Cio Limama, were charged with surviving victims, Marissa and Micel, the sums
double murder and double frustrated murder in an Amended of P50,000.00 and P30,000.00 as moral and exemplary
Information,3 the accusatory portion of which reads: damages, respectively.8

That on or about October 31, 1998, inthe Municipality of Ruling of the Court of Appeals
Kiblawan, Province of Davao del Sur, Philippines, and within
the jurisdiction of this Honorable Court, the above-named On appeal, the CA did not find any reason to disturb the
accused, conspiring, confederating and helping one another, findings of the RTC. However, it found merit in appellants
armed with assortedfirearms, with intent to kill with argument that the crime committed could not have been a
treachery and evident premeditation, did, then and there complex crime since the death and injuries complained of
willfully, unlawfully and feloniously, simultaneously strafe did not result from a single act but from several and distinct
the house of Eugenio Santander resulting to death of acts of shooting. And as treachery was alleged in the
[Cresjoy] Santander and RollySantander and seriously Amended Information and sufficiently proven during trial,
wounding Marissa Santander and Micel Santander, which appellants should be convicted instead of two counts of
would have caused their death had there been no timely and murder and two counts of frustrated murder. Thus, in
able medical assistance rendered to them, to the damage and rendering its Decision9 dated July 28, 2006, the CA disposed
prejudice of the offended parties. ofthe case as follows:

WHEREFORE, the judgment of conviction of appellants Jojo


Sumilhig, Alias Carding Sumilhig and Pasot Saloli is affirmed
ISSUE: butmodified as follows

WON: the crime committed is a complex crime Appellants Juan "Jojo" Sumilhig, Alias Carding Sumilhig and
Pasot Saloli, are found guilty beyondreasonable doubt of:

Rulings:
a. Murder for killing Crisjoy Santander, and x x x are True, the RTC noted in its Decision the existence of motive
sentenced to suffer the penalty of reclusion on the part of Jojo for committing the crime as well as
perpetua; Pasots incredulous claim of ignorance on almost about
everything. It is well to note, however, that the said court
b. Murder for the killing of Rolly Santander, and x x neither based the appellants conviction on the existence of
x are sentenced to suffer the penalty of reclusion such motive nor on Pasots weak defense of ignorance alone,
perpetua; but upon the prosecution witnesses identification of
appellants as the assailants, viz:
c. Frustrated Murder for the shooting of Marissa
Santander and x x x are sentenced to suffer an Assessing the evidence presented by both [P]rosecution and
imprisonment ofsix (6) years, four (4) months and defense, we see a less than glaring hint of vendetta. As part of
[ten] (10) days of prision mayoras minimum to his defense, the accused Jojo Sumilhig narrated that his
fourteen (14) years, eight (8) months and twenty family was massacred by Jerry Santander, brother of
(20) days of reclusion temporalas maximum; d. Remigio Santander [in] February 1998. Short of admitting
Frustrated Murder for the shooting of Micel the crime, Sumilhig stated that because of this, he harbored
Santander and x x x are sentenced to suffer an ill feelings not only against Jerry and Carlos Santander but
imprisonment ofsix (6) years, four (4) months and also against their family. Thus a clear motive for killing the
ten (10) days of prision mayoras minimum to Santander family has been established giving credence to
fourteen (14) years, eight (8) months and twenty prosecution witnesses allegation that after the strafing Jojo
(20) days of reclusion temporalas maximum. All Sumilhig shouted"Nakabalos na ko!" The likelihood of his
accused are ordered to indemnify jointly and intention to wipe out the said family became even more
severally the heirs of Crisjoy Santander and Rolly apparent.
Santander the sum of P100,000.00 and the
surviving victims Marissa Santander and Micel Despite his positive assertion that it was the Santanders that
Santander P50,000.00 as moral damages killed his family, he did not file any case against them. It was
and P30,000.00 as exemplary damages, without only after he was arrested that he filed a complaint against
subsidiary imprisonment in case of insolvency. Jerry and Carlos Santander.

Ruling of the Supreme Court His alibi likewise failed to meet the stringent requisites of the
Rules. Even as Dr. Quirapas appeared determined to rule out
The appeal has no merit. the possibility that he could walk without crutches five
Appellants conviction was based on months after his discharge, the same was based on general
their positive identification by the medical prognosis. Such prognostication admits certain
prosecution witnesses. exception[s], as could be gleaned from the testimony of the
doctor himself that the healing period may vary depending
on the age and physical condition of the patient. Notably There is no reason to doubt Jerry and Marios identification
Jojo Sumilhig was then 23 years old. of the appellants considering that (1) Jerry was just six
meters away from them;16 (2) the moon was bright and Jerry
What was certain was the positive identification made by was familiar with all the accused as most of them are his
Jerry Masaglang and Remegio Santander of all of the relatives;17 and, (3) Mario knows Jojoever since he was
accused. small.18 Besides, "[t]ime-tested is the rulethat between the
positive assertions of prosecution witnesses and the negative
The "overkill" by which the accused Pasot Saloli and Carding averments of the accused, the former undisputedly [deserve]
Sumilhig claimed total ignorance of almost anything only more credence and [are] entitled to greater evidentiary
served to arouse incredulity. Both accused claimed they weight."19
could not read, write, tell time, day, month or year. Neither
could [they] allegedly speak [or] understand Visayan, which Anent the respective alibis interposed by appellants, suffice it
is of common knowledge to be widely spoken in almost every to say that "[a]libi cannot prevail over the positive
part of Mindanao. Saloli claimed he did not know what day identification of a credible witness."20
[it] was when he was testifying, or the day before and after
that. Both claimed they did not know the complainants or of There was conspiracy among the accused.
the massacre that took place.
"[C]onspiracy exists when two or more persons come to an
xxxx agreement concerning the commission of a felony and decide
to commit it."21 It is not necessary to adduce evidence of a
More importantly, these claims [of] utter ignorance are previous agreement to commit a crime.22"Conspiracy may be
belied by the evasiveness by which all three accused shown through circumstantial evidence, deduced from the
answered in obvious effort to avoid criminal responsibility. mode and manner in which the offense was perpetrated, or
Behind the faade of ignorance and lack of education lurks a inferred from the acts of the accused themselves when such
calculating mind. We find [it] difficult to ascribe innocence leadto a joint purpose and design, concerted action, and
to the accused when traces of ingenuity and craftiness community of interest."23
characterize their testimonies.
Here, there is no proof of a previous agreement among the
All these observations however become insignificant in the accused but there is a series of events that clearly established
face of the positive and spontaneous identification of the conspiracy among them. First, they were all armed with
assailants/accused by credible witnesses Jerry Masaglang firearms. Second, they surreptitiously approached the crime
and Remigio Santander.15 scene. Third, when they were within close range of the
intended victims, they simultaneously discharged their
firearms. Fourth, they ceased firing at the same time and fled
together. Undoubtedly, their acts before, during and
immediately after strafing the house of Eugenio evince their Article 246 of the Revised Penal Code defines Parricide as
unanimity in design, intent and execution.24 Treachery follows:
attended the commission of the crime.
Art. 246. Parricide. - Any person who shall kill his father,
"There is treachery whenthe offender commits any of the mother, or child, whether legitimate or illegitimate, or any of
crimes against the person, employing means, methods his ascendants, or descendants, or his spouse, shall be guilty
of parricide and shall be punished by the penalty of reclusion
orforms in the execution thereof which tend directly and
perpetua to death.
specially to insure the execution, without risk to himself
arising from [any] defense which the offended party might Parricide is committed when: (1) a person is killed; (2) the
make."25 deceased is killed by the accused; and (3) the deceased is the
father, mother, or child, whether legitimate or illegitimate, or
Treachery is evident in this case as the suddenness and a legitimate other ascendant or other descendant, or the
unexpectedness of the assault deprived the victims of an legitimate spouse of the accused. The key element in
opportunity to resist it or offer any defense of their persons. Parricide - other than the fact of killing - is the relationship
This is considering that the victims were unaware that they of the offender to the victim. 23 All the elements are present in
would be attacked by appellants with a hailof bullets from this case. Jose, the victim, was killed by accused-appellant,
their firearms fired at close range. Indeed, "[t]he suddenness his own son.
of the attack, without the slightest forewarning thereof,
placed the [victims] x x x in such a position that they could The crime of Parricide is punishable by the indivisible
penalties of reclusion perpetua to death. With one mitigating
not have defended themselvesfrom the aggression x x x."26
circumstance, namely, voluntary surrender, and no
aggravating circumstance, the imposition of the lesser
The crime committed is two counts of penalty of reclusion perpetua on accused-appellant was
murder and two counts of frustrated proper.
murder.

FACTS:
PEOPLE OF THE PHILIPPINES, Plaintiff- The said accused, did then and there willfully, unlawfully and
Appellee, v. ADRIAN GUTING Y TOMAS, Accused- feloniously, and with evident premeditation, that is, having
Appellant. conceived and deliberated to kill his own father Jose Guting
G.R. No. 205412, September 09, 2015 LEONARDO- y Ibarra, 67 years old, married, while inside their residential
DE CASTRO, J. house, and armed with a bladed weapon, suddenly and
unexpectedly stabbed several times the victim, employing
DOCTRINE: means, manner and form in the execution thereof which
tender directly and specially to insure its commission
without danger to the person of said accused, the result of
which attack was that said victim received multiple stab collectively, pointed to accused-appellant as the perpetrator
wounds on his body which directly caused his instantaneous of the brutal killing of his father.
death.
ISSUE:
The RTC promulgated its Decision on June 24, 2010 finding Whether the accused-appellant guilty beyond reasonable
accused-appellant guilty of Parricide based on his verbal doubt of the crime charged despite the prosecution's failure
admission that he killed his father, Jose. Even assuming that to overthrow the constitutional presumption of innocence in
accused-appellant's admission was inadmissible in evidence, his favor.
the RTC adjudged that the prosecution was still able to
establish sufficient circumstantial evidence which, taken

You might also like